100 Free Practice OMM Questions


Best Practice OMM Questions


100 Free Practice OMM Questions


OMM Questions 1-25


1| A patient presents with a neurological deficit resulting in an absent triceps tendon reflex and weakness in the extensors of the hand, wrist and elbow. Which nerve is most likely involved?

A) Musculocutaneous
B) Median
C) Radial
D) Ulnar
E) Axillary

Answer

Answer C

The radial nerve innervates the extensors of the upper extremity [elbow (triceps), wrist and hand]. An absent triceps (C7) reflex and wrist drop will indicate radial nerve pathology.

The musculocutaneous nerve innervates the biceps, brachialis, and coracobrachialis. A patient with a musculocutaneous nerve injury presents with a loss of the biceps reflex (C5, C6). Answer A

The median nerve innervates forearm pronators, wrist and finger flexors and thenar muscles. Answer B

The ulnar nerve innervates flexor carpi ulnaris, hypothenar muscles and the adductor pollicis. Answer D

The axillary nerve innervates the teres minor and deltoid muscles. Injury to the axillary nerve results in atrophy of the deltoid and weakness with shoulder abduction and flexion. Answer E


2| A 47-year-old male with a history of diabetes comes to the emergency department with acute onset of low back pain. The pain started suddenly when he was sliding into home plate during a softball game earlier today. The pain is severe and radiates into his right lower extremity. He describes an electric type pain shooting into his right foot. What are some findings you would expect to see when examining him?

A) Edematous, boggy tissue with palpation, and full active range of motion of the lumbar spine. Decreased sensation on the dorsum of the foot.
B) Severe, sharp pain with palpation of lumbar tissues, guarded range of motion of the lumbar spine, and increased deep tendon reflexes of the right lower extremity.
C) Warm tissue texture changes, hypertonic muscles, and decreased range of motion of the lumbar spine.
D) Little to no edema or erythema, cool dry skin to palpation, ropy and fibrotic muscles, and decreased range of motion of the lumbar spine.
E) Dull pain with palpation. Positive straight leg raising test

Answer

Answer C

The above patient has an acute somatic dysfunction of the lumbar spine. He also herniated one of his lumbar discs. Acute somatic dysfunctions are associated with severe sharp pain (especially with palpation). Acute tissue texture changes are usually described as edematous, erythematous, and boggy. There is increased moisture on the overlying skin and associated hypertonic muscles. The muscle contraction and guarding will cause a limited range of motion of the involved areas.

Answer A did not contain the limited range of motion in the lumbar spine that is consistent with this type of injury. Answer A

An acute herniated disc results in decreased reflexes of the effected lower extremity. Answer B

This patient likely has a positive straight leg raising test and decreased sensation somewhere on the foot, secondary to a herniated disc. Answers D and E included these findings however, they were coupled incorrectly with descriptors of a chronic somatic dysfunction. Chronic tissue texture changes include cool dry skin; flaccid, doughy, or mushy muscles; and firm, ropy, thickened, fibrotic interstitial tissues. Decreased range of motion is secondary to fibrotic tissues or possibly contractures. Answers D and E


3| A 55-year-old male is complaining of low back pain. On examination you notice transverse process of L5 is posterior on the right. Extending L5 worsens the asymmetry while flexion restores rotational symmetry. Which one of the following is the best statement regarding this patient’s somatic dysfunction?

A) Due to the non-neutral L5 somatic dysfunction, a group dysfunction of the high lumbar region will be present.
B) This patient has a right-sided psoas syndrome.
C) L5 will resist left rotation in the extended position.
D) When employing a muscle energy (post-isometric relaxation) technique the patient would be asked to rotate his torso to the left against isometric contraction.
E) Extending L5 will greatly limit its side-bending to the right

Answer

Answer C

The above patient has a lower extremity that is awaiting surgical stabilization. The rhythmic motion of the pedal pump will exert forces at the still unstable fracture site.

There are several different techniques to release the thoraco-abdominal diaphragm. Some are very gentle and can be performed on individuals with several concurrent medical or surgical conditions. Answer A

Thoracic rib raising and paraspinal inhibition will not harm the recent surgical site in the cervical spine or the lower extremity fracture. Answer B

Although myofascial release and counterstrain would be contraindicated in some areas for the above individual, the modalities are not specifically contraindicated and thus not the best answers. Answers D and E


4| In a patient with low back pain, the dysfunctional T12 segment is found to have restriction in a transverse plane and around a transverse axis. Which of the following dysfunctions best describes the position of T12?

A) T12 is flexed
B) T12 is sidebent right
C) T12 is rotated left
D) T12 is neutral sidebent right, rotated left
E) T12 is extended rotated right, sidebent right

Answer

Answer E

Vertebral motion around a transverse axis is flexion or extension. So therefore T12 must be either flexed or extended. Vertebral motion in a transverse plane is rotation. So therefore T12 must also be rotated. Understanding Fryette’s principles one must conclude if a vertebrae is flexed or extended and rotated it must be sidebent toward the same side.

T12 has restricted motion around a transverse axis, therefore it is flexed or extended, not neutral. Answer D

Answers A, B and C only comment on one plane of motion.


5| In a patient with neck pain, the atlas is rotated to the left. Given this information what else would you expect to find on this patient?

A) The OA joint will resist translation to the left.
B) C2 will resist anterior glide on the right
C) C2 will resist right sidebending
D) The lower cervical segments will be rotated to the right to compensate for the somatic dysfunction of this upper segment
E) With the patient’s neck flexed to 45 degrees, he will be able to turn his head further left than right.

Answer

Answer E

Rotation of the atlas (on the axis) is the AA joint. Flexing the patient’s head will lock out rotation of C2 – C7 and will isolate rotation to the AA joint. Since the AA joint is rotated to the left, the patient should be able to turn his head further left than right.

The AA joint will not necessarily affect motion of the occiput on the atlas (OA joint) or C2 on C3. Answers A, Band C

Motion at the AA joint will not necessarily cause compensation or somatic dysfunction of the lower cervical units. Answer D


6| 22-year-old male presents with low back pain following a fall onto a concrete floor. The patient gives a history of episodic aching of the lumbar region prior to the fall. Examination of the patient in the prone position reveals a deep sacral sulcus on the left, a posterior/inferior ILA on the right when compared to the opposite side, and a lumbosacral junction that springs freely upon compression. The most likely diagnosis is?

A) A forward sacral torsion on a right oblique axis
B) A forward sacral torsion on a left oblique axis
C) A backward sacral torsion on a left oblique axis
D) A left unilateral sacral flexion
E) Bilateral sacral extension

Answer

Answer A

In a patient with a deep left sulcus and a negative lumbosacral spring test the left portion of the sacrum has moved anterior. Since the above patient has a posterior/inferior ILA on the right this indicates that this portion of the sacrum has moved posteriorly. A forward sacral torsion on a right oblique axis (Le. a right rotation on a right oblique axis) is the only answer that would be consistent with the above findings.

In a forward sacral torsion on a left oblique axis (left on left) the right sulcus would be deeper and the left ILA would be posterior/inferior. Answer B

In a backward sacral torsion or a bilateral sacral extension, the lumbosacral spring test would be positive (Le. the lumbosacral junction would not spring). Answers C and E

In a unilateral sacral flexion on the left, the left ILA would be posterior and significantly inferior. Answer D


7| 5-year-old child with asthma is brought to the ED by his father. The child developed an upper respiratory tract infection 48 hours ago. Over the past hour his breathing has become increasingly labored and he has started to develop respiratory distress. His father has administered one albuterol treatment via a nebulizer one hour ago; there was slight improvement in the patient’s symptoms. On examination, respiratory rate =50. Inspection of the chest wall reveals subcostal retractions and evidence of accessory muscle use. Which of the following can be done initially to improve the patient’s symptoms?

A) Oxygen, oral albuterol, intravenous corticosteroids, and thoracic manipulation to stimulate sympathetic.
B) Oxygen, inhaled albuterol, intravenous corticosteroids and seated thoracic pump technique.
C) Oxygen, inhaled albuterol, zafirlukast, and manipulation to the cervical spine to normalize vagal tone.
D) Oxygen, oral corticosteroids, and HVLA to rib 3.
E) Oral albuterol, cromolyn sodium, and CV4 technique.

Answer

Answer B

Acute management of asthma includes supplemental oxygen, inhaled bronchodilators (albuterol) and intravenous corticosteroids. Osteopathic techniques that have been used successfully for asthma include the seated thoracic pump technique, CV4 technique, and cervical spine manipulation
to normalize the vagus.

Although the patient did not respond to one treatment of inhaled albuterol, additional doses can be administered every 20 minutes. Oral albuterol is not recommended if inhaled medications can be administered. Answer A

Zafirlukast is an oral leukotriene receptor agonist. It is useful for patients with mild to moderate symptoms, however it’s onset of action is delayed for approximately 2 weeks. Answer C

Oral corticosteroids may be effective, however the preferred route of administration is IV. The asthmatic patient will usually present with a rib 3 or 4 dysfunction, however, as with most viscero-somatic reflexes, it responds poorly to HVLA. Answer D

Cromolyn sodium is a medication that is useful in patients with mild asthma, especially allergic asthma. However, onset of action is delayed several weeks. Answer E


8| The first cervical segment has the greatest degree of freedom in which plane(s)?


A) Transverse
B) Coronal
C) Sagittal
D) Oblique
E) Transverse and Coronal

Answer

Answer A

Motion in the transverse plane is rotation. Segmental motion is conventionally defined as the specified segment on the one below. Since rotation is the main motion of the first cervical segment (the AA joint-the atlas on the axis) it has the greatest degree of freedom in the transverse plane.

Motion in the coronal plane is sidebending. Answer B

Motion in the sagittal plane is flexion/extension. Answer C

The oblique plane is not a true description of a plane. Answer D

Vertebral motion in the transverse and coronal planes refer to rotation and sidebending respectively. There is a slight amount of flexion allowed at the atlas, but no sidebending. The AA joint essentially allows rotation only. Answer E


9| Which of the following is common in lumbar stenosis with neurogenic claudication, but is not typical in peripheral vascular disease with vascular claudication?

A) Pain relief with sitting
B) Increased pain when treating a typical posterior lumbar tender point
C) Absence of lower extremity weakness
D) Increased lower extremity pain with walking
E) Posterior calf pain

Answer

Answer B

Spinal extension decreases the size of the intervertebral foramina and it decreases the AP diameter of the spinal column. Spinal extension is necessary when treating a typical posterior lumbar tender point. Extension increases pain associated with spinal stenosis, but will not affect the patient with vascular claudication since the patient is at rest.

Sitting flexes the spinal column, thus decreases pain associated with neurogenic claudication. Sitting (resting) will also decrease the symptoms associated with vascular claudication. Answer A

Lower extremity weakness is seen in patients with neurogenic claudication, but not in patients with vascular claudication. Answer C

Symptoms of lumbar spinal stenosis (pain, numbness and sometimes weakness in the lower extremities) typically increased with extension of the spinal column (standing, walking). Symptoms of peripheral vascular disease, (leg pain and cramping) increases with activity and decreases with rest. Walking produces symptoms in both vascular and neurogenic claudication. Answer D

Posterior calf pain is common in both neurogenic and vascular claudication. Answer E


10| Which of the following is considered to be the most common congenital anomaly in the lumbar spine?


A) Sacralization
B) Lumbarization
C) Facet hypertrophy
D) Facet tropism
E) Spina bifida occulta

Answer

Answer D

Facet tropism is a misalignment of the facet joint. According to Osteopathic Principles in Practice it is the most common anomaly of the lumbar spine.

Sacralization occurs in approximately 3.5% of the population. Answer A

Lumbarization occurs in <1 % of the population. Answer B

Facet hypertrophy is not a congenital anomaly. It is a condition often seen with degenerative changes. Answer C

Spina bifida occulta occurs in about 10% of the population. Answer E


11| A 38-year-old mildly obese patient comes to your office complaining of buttock pain. She fell on her buttocks about one week ago and she still has some residual pain. Inspection of her lumbar spine and gluteal region reveals a decreased lumbar lordosis. Standing flexion test and seated flexion test show no evidence of asymmetry. Sacral sulci are equal in depth. The lumbosacral junction does not spring. Based on the information given what is the most likely diagnosis?

A) Bilateral sacral extension
B) Forward sacral torsion on a left oblique axis
C) Anterior sacral base
D) Unilateral extended sacrum on the right
E) Unilateral extended sacrum on the left

Answer

Answer A

The above patient has a bilateral sacral extension (a.k.a. sacral base posterior or a posterior sacral base). The examiner will notice that the sacral sulci are equal in depth and there is no springing at the lumbosacral junction (i.e. the lumbosacral spring test is positive). The seated flexion test will be FALSELY negative – Why? Since both SI joints are restricted, asymmetry will not be appreciated.

In a forward sacral torsion (Ieft-on-Ieft or right-on-right) the seated flexion test would be positive, the sulci would appear asymmetric, and the lumbosacral spring test would be negative. Answer B

In an anterior sacral base (a.k.a. bilateral sacral flexion or sacral base anterior), the findings will be the same, except that the lumbosacral junction would spring (lumbosacral spring test is negative). Answer C

In a unilateral extended sacrum on the right the seated flexion test will be positive on the right, the right sulcus will appear shallow (posterior) and the lumbosacral spring test will be positive. Answer D

In a unilateral extended sacrum on the left, the seated flexion test will be positive on the left, the left sulcus will appear shallow (posterior) and the lumbosacral spring test will be positive. Answer E


12| In a patient with acute onset of low back pain, structural examination reveals a positive seated flexion test on the right. The left sacral base is anterior when compared to the right. The spring test is positive. Given the above information what is the most likely sacral diagnosis?

A) Left sacral rotation on a right oblique axis
B) Right sacral rotation on a right oblique axis
C) Right sacral rotation on a left oblique axis
D) Right unilateral sacral flexion
E) Extended sacral base

Answer

Answer C

In a sacral torsion (or sacral rotation on an oblique axis) the seated flexion test is positive on the opposite side of the axis. In this case the right positive seated flexion test indicates a left oblique axis. A positive spring test (a.k.a. lumbosacral spring test) indicates that part of the sacral base has moved posterior. Since the left sacral base is anterior (i.e. the left sulcus is deep), then this must indicate that the right sacral base has moved posterior. Out of all the answers listed the only possible one that would correlate with the above findings is right sacral rotation on a left oblique axis.

In left sacral rotation on a right oblique axis, and right sacral rotation on a right oblique axis there would be a positive seated flexion test on the left. Answers A and B

In a right unilateral sacral flexion the right sulcus would be deeper, and the lumbosacral junction would spring freely. Answer D

In an extended sacral base the seated flexion test would be falsely negative, and the sulci would appear symmetric. Answer E


13| A patient has a sacral torsion that resulted in a right anterior sacral base and a negative lumbosacral spring test. Based on the information given, what is the expected somatic dysfunction at L5?


A) L5 NSLRR
B) L5 NSRRL
C) L5 ERRSL
D) L5 FRLSL
E) L5 FRRSL

Answer

Answer A L5 NSLRR

If a sacral torsion is present, a right sacral base anterior (a deep right sulcus) and a negative lumbosacral spring test indicate a left on left torsion. Using the rules of L5 on the sacrum and Fryette’s principles one can figure out the dysfunction of L5.

Rules of L5:
1) When L5 is rotated, the sacrum rotates in the opposite direction.
– Sacrum rotated left, L5 must be rotated right.

2) When L5 is sidebent, a sacral oblique axis is engaged on the same side as the sidebending.
– Sacrum has a left oblique axis, L5 must be sidebent left.

Fryette’s principle I:
If L5 is rotated right and sidebent left, L5 must be in the neutral plane (L5 NSLRR).


14| A 49-year-old male presents to your office complaining of chronic lumbosacral pain. There is a negative standing flexion test and a positive seated flexion test. Structural examination reveals a deep sacral sulcus on the left, posterior/inferior ILA on the right, and the lumbosacral junction springs freely upon compression. The most likely diagnosis is:

A) Right sacral rotation on a right oblique axis
B) Left sacral rotation on a left oblique axis
C) Right sacral shear
D) Right sacral rotation on a left oblique axis
E) Left sacral rotation on a right oblique axis

Answer

Answer A

Right sacral rotation on a right oblique axis. A right sacral rotation on a right oblique axis will result in a deep sacral sulcus on the left, a posterior/inferior ILA on the right, and the lumbosacral junction springs freely upon compression.

A left rotation on a left oblique will result in a deep sacral sulcus on the right and a posterior/inferior ILA on the left. Answer B

A right sacral shear can either be a right unilateral sacral flexion or a right unilateral sacral extension. A right unilateral sacral extension will result in a positive lumbosacral spring test. A right unilateral sacral flexion will result in a deep sacral sulcus on the right (not left). Answer C

A backward sacral torsion (right-on-Ieft or a left-on-right) will result in a positive lumbosacral spring test (Ie. the lumbosacral junction will not spring freely). Answers D and E


15| A 47-year-old female with new onset left upper extremity weakness, miosis, anhydrosis and ptosis was seen by her primary care physician two weeks ago. Diagnostic work-up revealed a Pancoast tumor in the left upper lobe. The tumor appears to be invading the inferior trunk of the brachial plexus. This is the likely etiology of her upper extremity weakness. Based on the tumor’s location in the brachial plexus, what physical exam findings would you expect this patient to have?

A) Winging of the scapula
B) Weakness with finger abduction
C) Weakness with arm abduction
D) Weakness with elbow flexion
E) Numbness of the lateral aspect of the forearm

Answer

Answer B

The inferior trunk is formed by the C8 and T1 nerve roots. As a result, this will produce weakness with the intrinsic muscles of the hand (interossi) and deep finger flexors. It will also likely produce numbness in the little finger (C8) and medial elbow (T1).

The winging of the scapula is due to injury to the long thoracic nerve, which is formed by the C5 – C7 nerve roots. Answer A

The deltoid and supraspinatus is primarily responsible for arm abduction. The supraspinatus (suprascapular nerve) receives innervation from the superior trunk. The deltoid (axillary nerve) receives innervation from posterior cord, specifically from C5 and C6 nerve roots. The axillary nerve does not receive contributions from the inferior portion of the brachial plexus. Answer C

Numbness at the lateral aspect of the forearm and elbow flexion (musculocutaneous nerve) would not be affected since it stems from the superior trunk of the brachial plexus. Answers D and E


16| Nerve entrapment at Guyon’s canal is likely to produce?


A) Wrist adduction weakness
B) Wrist flexion weakness
C) Finger abduction weakness
D) Thumb abductor weakness
E) Weakness with flexion of the 4th and 5th DIP’s

Answer

Answer C

Guyon’s (pisohamate) canal is located at the wrist. The medial border is the pisiform bone, lateral border is the hamate, roof is the flexor retinaculum and floor is the pisohamate ligament. The ulnar nerve runs through the Guyon’s canal. Nerve entrapment would produce weakness in the hypothenar muscles, adductor pollicis, interossei, and 3rd and 4th lumbricals. The interossei are responsible for abduction and adduction of the fingers.

Wrist adductors and wrist flexors originate in the forearm and would not be affected with entrapment at the wrist. Answer A and B

The median nerve innervates the abductor pollicis brevis. The radial nerve innervates the abductor pollicis longus. Answer D

Weakness with flexion of the 4th and 5th DIP’s (ulnar portion of flexor digitorum profundus) would be due to injury to the ulnar nerve proximal to the wrist (most likely at the cubital tunnel). Answer E


17| A 65-year-old female fell on some ice while walking into her house. She slipped backward landing on her buttock as well as the palm of her hand. She fell in such a way that her right forearm was supinated and her shoulder, elbow and wrist were extended. Based on the above mechanism of injury, which dysfunction is most likely to be seen in this patient?

A) Left on Left sacral torsion
B) Right on Right sacral torsion
C) Right anterior radial head somatic dysfunction
D) Right posterior radial head somatic dysfunction
E) Right adducted ulna

Answer

Answer C

The patient has an anterior radial head somatic dysfunction. Radial head somatic dysfunction can often be caused by a fall on an outstretched arm and can result in wrist and elbow pain. Specifically, an anterior radial head somatic dysfunction is usually caused by a fall on an outstretched supinated arm. Supination of the wrist and forearm physiologically causes the radial head to glide anteriorly.

A left on left torsion as well as a right on right torsion can be due to trauma, however “falling on the buttocks” is too nonspecific to indicate either dysfunction. Answers A and B

A posterior radial head somatic dysfunction can be due to a fall forward on an outstretched pronated arm. Pronation of the wrist and forearm physiologically causes the radial head to glide posteriorly. At the impact of the fall, the arm is positioned with the shoulder flexed, elbow and wrist extended and the forearm pronated. Answer D

Abducted ulna is not associated with trauma from falling backward on an outstretched arm. Answer E


18| Which of the following statements is true regarding the biomechanics of the lower extremity?

A) Anterior glide of the talus provides talocrural stability
B) The head of the femur will glide posterior with external rotation of the hip
C) Patella femoral syndrome has been associated with an increase of the femoral neck angle.
D) Restriction within the subtalar joint will limit internal and external rotation of the leg while the foot is fixed
E) Somatic dysfunctions of the ankle most commonly occur when the foot is dorsiflexed

Answer

Answer D

The subtalar joint acts mostly as a shock absorber and allows internal and external rotation of the leg while the foot is fixed. Restriction within this joint will limit this motion.

The talocrural joint (a.k.a. tibiotalar joint) is composed of the articulation of the trochlea of the talus and the ankle mortise. Posterior glide of the talus (dorsiflexion) provides stability to this joint. This joint is relatively unstable in plantar flexion (anterior glide of the talus), and allows the greatest degree of inversion/eversion in this position. Answer A

The head of the femur will glide anterior with external rotation of the hip, and posterior with internal rotation of the hip. Answer B

It is believed that an increase in the Q angle is associated with patellofemoral syndrome (a.k.a. lateral patella femoral tracking syndrome). Changes in this angle can result in genu valgum or genu varum. Changes in the femoral neck angle are associated with coxa valga and coxa vara. Answer C

Somatic dysfunctions of the ankle most commonly occur when the foot is supinated because of the relative instability of the ankle in this position. Answer E


19| Which of the following statements is true of a left cranial torsion of the sphenobasilar synchondrosis?

A) The sphenoid rotates about an AP axis so that the left greater wing is inferior
B) The sphenoid and occiput will rotate in the opposite directions about an AP axis
C) The sphenoid will rotate left about a vertical axis
D) The occiput will rotate left about a vertical axis
E) The sphenoid will rotate anterior about a transverse axis

Answer

Answer B

In a cranial torsion of the SBS (which is the same as a cranial torsion- just a different way of saying it) the sphenoid and occiput rotate in opposite directions about an AP axis. Specifically in a left cranial torsion, the sphenoid will rotate so that the left greater wing is superior.

In a left cranial torsion, the sphenoid will rotate so that the left greater wing of the sphenoid is superior,
not inferior. Answer A

The sphenoid and occiput rotate about vertical axes in lateral strains. Answers C and D

The sphenoid will rotate about a transverse axis with flexion and extension phases of the CRI. Answer E


20| A 15-year-old female is experiencing low, midline, wave-like cramping pelvic pain that occurs with menses. The pain has been present for 3 months and has been getting progressively worse. Pain frequently radiates to her lumbar spine and is associated with frequent nausea. In addition to appropriate oral medications, which osteopathic manipulative technique will most effectively decrease the patient’s symptoms by altering sympathetic tone?

A) Rib raising and paraspinal inhibition to the thoracoLumbar junction
B) Celiac ganglion release
C) Pelvic diaphragm release
D) Muscle energy to T1 0
E) Sacral inhibition

Answer

Answer A


The above patient has signs and symptoms of dysmenorrhea. Pain is produced by uterine vasoconstriction, anoxia, and sustained contractions mediated by prostaglandins. The uterus receives sympathetic innervation from T10 – L2 segments. Rib raising and paraspinal inhibition will decrease sympathetic tone, therefore these techniques will most effectively enhance blood flow and relax the uterus, thus decreasing pain.

The purpose of the celiac ganglion release is to calm sympathetics from T5 – T9. This technique will not affect autonomic tone to the uterus. Answer B

The pelvic diaphragm release will not effectively normalize sympathetic tone in the uterus. Answer C

Although manipulative techniques at vertebral levels could alter autonomic function, muscle energy at T10 will not greatly affect sympathetic tone in the uterus because treatment of one segment is less likely to be as effective as compared to treatment of a few levels. Answer D

Parasympathetic flow to the uterus originates in the pelvic splanchnic nerves (S2-S4). Treatment directed at the sacrum will influence parasympathetic tone, not sympathetic tone. Answer E


21| A 67-year-old female comes to your office with frequent burning substernal and epigastric pain. The pain is associated with fatty meal intake and is relieved with over the counter antacids. The pain has been present for one year but has gotten worse over the past several weeks. Based on the above diagnosis, palpatory changes are likely to arise at which of the following vertebral levels?

A) C5
B) T3
C) T6
D) T10
E) T12

Answer

Answer C

This patient is likely to have a gastric ulcer or GERD. Unlike duodenal ulcers, gastric ulcers symptoms typically worsen after eating.

Abnormal stimuli from the gastric mucosa will enter the spinal cord at the T5 – T9 level and facilitate these segments. This may result in abnormal efferents to somatic structures leading to palpatory changes at these levels.

Since C5 is not closely associated with autonomic innervation to the upper GI tract. Answer A

Efferents from T3 affects sympathetic tone in the heart, lung and head. Answer B

In the Gl system T10 – T11 is associated with sympathetic tone in portions of the duodenum and the pancreas, jejunum, ileum and proximal 2/3 of transverse colon. Answer D

In the Gl system T12 – L2 is associated with sympathetic tone in the distal 1/3 of the transverse colon and rectum. Answer E


22| A 48-year-old diabetic male has chronic renal insufficiency. Structural examination is likely to reveal:


A) Ropy paraspinals at C3
B) Severe sharp tenderpoints at T5 and T6
C) Fibrotic paraspinals T8 and T9
D) Ropy paraspinals at T1 0 – L1
E) Fibrotic tissue texture changes at the sacrum

Answer

Answer D

Renal insufficiency has been associated with tissue texture changes at the thoracolumbar junction. Key terms that describe chronic tissue texture changes are ropy and fibrotic.

The parasympathetics, which supply the kidney and upper ureter, originate from the vagus nerve and are influenced by the occiput, C1 and C2, not C3.

Although it is not known how the parasympathetics affect the kidney, they do maintain normal peristaltic waves in the ureters. Answer A

T5 and T6 are not segmentally related to the kidney. Answer B

Sympathetic fibers arising from T8 – T9 do not supply the kidney. Answer C

Parasympathetics from the pelvic splanchnic supply the lower ureter and bladder. Answer E


23| An 84-year-old male with chronic peptic ulcer disease may have changes related to viscerosomatic reflexes in which of the following areas?


A) T1 – T4
B) T5 – T7
C) T9 – T11
D) T11 – L2
E) L3 – L5

Answer

Answer B

Sympathetic control for the upper GI tract is via TS – T9 visceral efferents.

Abnormal stimuli from visceral afferents enter the spinal cord, sensitize interneurons and produce an exaggerated motor response (a process called facilitation) at these segments.

Head and neck =T1 – T4. Answer A

T9 – L2 = left colon and pelvic structures. Answers C and D

L3 – L5 = Nothing!!! Answer E


24| A 29-year-old female in her 36th week of pregnancy is having some leg edema. The most effective initial treatment is?


A) Pedal pump
B) Thoracic inlet release
C) Rib raising to the thoracoLumbar junction
D) Pla
ce the patient in the Trendelenburg position
E) Muscle energy to L1 – L3

Answer

Answer B

Prior to lymphatic pumping techniques, the thoracic inlet should be released to remove any impedance into the thoracic duct and ultimately the central circulation.

The thoracic inlet should be released first before a lymphatic pump is attempted. Answer A

Edema in the lower extremities is due to the mechanical effect of the uterus and abdominal contents pushing against the inferior vena cava. Rib raising to the thoracoabdominal junction will normalize sympathetics, but in the above case sympatheticotonia (increased sympathetic tone) is not the cause of the edema. Answer C

Placing the patient in the Trendelenburg position will increase the pressure against the diaphragm and may make the patient short of breath. Therefore, it is not appropriate. Answer D

Muscle energy will do little to improve this patient’s lower extremity edema. Answer E


25| 39-year-old female complains of abdominal pain that is associated with anorexia, fatigue, and weakness. Physical exam reveals an area of hyperpigmentation in the palmar creases, and sparse axillary hair. Lab results reveal low sodium, and elevated potassium, and calcium levels. At what spinal level would you expect to find tissue texture changes due to a viscerosomatic response from the above condition?

A) T2
B) T4
C) T8
D) T10
E) L2

Answer

Answer D

This patient has Addison’s disease, which is a disease of the adrenal gland and would cause a viscerosomatic response at T10.

T2 and T4 receive sympathetic innervation from the head and neck, heart esophagus and lungs, not the adrenal medulla. Answers A and B

T8 would receive sympathetic innervation for the upper GI tract. Answer C

L2 would receive innervation from the lower GI tract, uterus and cervix, penis, clitoris and legs. Answer E



OMM Questions 26-30


26| Which of the following techniques best evaluates inferior motion of the clavicles at the sternum?

A) Patient prone, index fingers on the anterior aspect of the clavicular heads near sternum, patient shrugs shoulders
B) Patient supine, index fingers on the superior aspect of the clavicular heads near sternum, patient depresses the shoulders
C) Patient supine, index fingers on the superior aspect of the clavicular heads near sternum, patient shrugs shoulders
D) Patient supine, index fingers on the anterior aspect of the clavicular heads near sternum, patient protracts the shoulders
E) Patient supine, index fingers on the anterior aspect of the clavicular heads near sternum, patient retracts the shoulders

Answer

Answer C

The clavicle moves in three different planes. First, it moves superior and inferior with shrugging and depressing the shoulder. second, it moves anterior and posterior with retraction and protraction of the shoulder. Third, it rotates anteriorly and posteriorly with internal and external rotation of the arm when it is abducted at 90 degrees

Shrugging the shoulder causes the lateral end of the clavicle to move superior, this causes the medial end to move inferior, via a see-saw motion.

This motion is easily observed with the patient in the supine position.

The clavicle cannot be properly evaluated in the prone position. Answer A

Patient supine, index fingers on the superior aspect of the clavicular head near sternum, patient depresses the shoulders. This will move the clavicle superior at the sternum, if no somatic dysfunction is present. Answer B

Patient supine, index fingers on the anterior aspect of the clavicular head near sternum, patient protracts the shoulders. This will move the clavicle posterior at the sternum, if no somatic dysfunction is present. Answer D

Patient supine, index fingers on the anterior aspect of the clavicular head near sternum, patient retracts the shoulders. This will move the clavicle anterior at the sternum, if no somatic dysfunction is present. Answer E


27| 63-year-old female fell off a chair and landed on her buttocks. She complains of severe, unrelenting low back pain. There is point tenderness to percussion of the spinous process of L3. A lumbar spine X-ray confirms your diagnosis. The most likely etiology of the pain is:

A) Tumor
B) Herniated nucleus pulposus
C) Cauda equina syndrome
D) Spinal stenosis
E) Compression fracture

Answer

Answer E

This is the classic presentation of a compression fracture. The key findings are spinous process point tenderness to percussion with confirmation on X-ray.

X-rays primarily identify bony and some soft tissue problems.

A tumor would usually be painless, not result from trauma, and it is unlikely to be visible on X-ray unless it obscures a structure or displaces it. Answer A

Herniated nucleus pulposus and cauda equina syndrome would not be visible on X-ray, and do not fit the case history. Answers B and C

Spinal stenosis would be visible on X-ray, but pain is not usually localized to the spinous process, or elicited with percussion of the spinous process. Answer D


28| 32-year-old female complains of dysuria and odd smelling urine. Physical examination reveals suprapubic tenderness. At what spinal levels would you expect to find the tissue texture changes related to a viscerosomatic reflex that would confirm the diagnosis?

A) T2-T4
B) T5-T7
C) T8-T10
D) T11-L1
E) L2-L5

Answer

Answer D

This patient most likely has a urinary tract infection, and would have tissue texture changes from a viscerosomatic reflex at Tll-Ll.

T2-T4 receives sympathetic innervation from the head and neck. Answer A

T5-T7 receives sympathetic innervation from the upper GI tract. Answer B

T8-Tl0 receives sympathetic innervation from the upper and middle GI tract. Answer C

The spinal cord stops at L2; therefore the lowest possible segment to receive sympathetic innervation is L2. Answer E


29| A patient who recently had a parathyroid adenoma removed should have what spinal level examined for a viscerosomatic reflex?


A) T1-T3
B) T5-T6
C) T7-T9
D) T10-T12
E) T12-L2

Answer

Answer A

The parathyroid glands are structures in the neck, and could have caused viscerosomatic reflex changes at the spinal level T1-T4.

T5-T6 spinal levels are associated with the viscerosomatic reflex from the heart, lungs and esophagus. Answer B

T7-T9 spinal levels are associated with the viscerosomatic reflex from the upper GI tract. Answer C

T10-T12 spinal levels are associated with the viscerosomatic reflex from the middle GI tract and the gonads. Answer D

T12-L2 spinal levels are associated with the lower GI tract. Answer E


30| While evaluating a mid thoracic somatic dysfunction, your left thumb is more anterior than your right. Your thumbs become symmetric when the patient is in flexion, however you cannot examine the patient in extension because the pain is too great. The most likely diagnosis is?

A) FRLSL
B) ERLSL
C) NRRSL
D) FRRSR
E) ERRSR

Answer

Answer D

Flexed, rotated right, sidebent right is the correct diagnosis. On the board exams it may be abbreviated as FRRSR.

The static evaluation in the neutral position shows that the left thumb is more anterior, which means the segment is rotated right.

Since the asymmetry at the segment resolved with flexion, the freedoms of motion are flexion, right rotation and left sidebending.

In a FRLSL (flexed rotated left, sidebent left) somatic dysfunction your left thumb would be posterior, not anterior. Answer A

In an EFLSL (extended rotated left, sidebent left) somatic dysfunction your left thumb would be posterior, not anterior, and symmetry would be restored in extension. Answer B

In a NRRSL (neutral, rotated right, sidebent left) somatic dysfunction symmetry would not be restored in flexion. Answer C

In an E~SR (extended, rotated right sidebent right) somatic dysfunction symmetry would be restored in extension. Answer E


31| A patient is diagnosed with a right anterior innominate rotation. About which sacral axis does this rotation occur?


A) Inferior transverse
B) Oblique
C) Middle transverse
D) Sagittal
E) Superior transverse

Answer

Answer A

The innominates rotate about an inferior transverse sacral axis with ambulation, and in posterior and anterior innominate somatic dysfunctions.

Sacral rotation occurs about 2 oblique axes with ambulation and with sacral torsions. Answer B

Postural motion occurs about a middle transverse sacral axis. Answer C

Sacral margin somatic dysfunctions rotate about a vertical axis. Answer D

Respiration and craniosacral motion occur about the superior transverse sacral axis. Answer E


32| A 25-year-old female patient complains of progressively worsening low back pain. The pain is located at the lumbosacral junction. Two days ago, she vaginally delivered a 9lbs. 20z baby boy. On examination the seated flexion test is negative. The most likely diagnosis is:

A) Left on left sacral torsion
B) Bilateral sacral flexion
C) Right innominate anterior
D) Bilateral sacral extension
E) Right inferior pubic shear

Answer

Answer B

A bilateral sacral flexion occurs during the delivery phase of childbirth when the sacrum nutates (sacral base moves anteriorly) to allow more space for the fetus to pass through the pelvic outlet into the birth canal. If the sacrum fails to counter-nutate (sacral base moves posteriorly), a bilaterally sacral flexion dysfunction remains. Due to birth mechanics, bilateral sacral flexion is a common dysfunction in the postpartum patient. Since the sacrum is bilaterally flexed it would
cause a falsely negative seated flexion test.

A left on left sacral torsion would have a positive seated flexion test on the right. Answer A

Right innominate anterior and a right inferior pubic shear are not specifically associated with childbirth. Answers C and E

A bilateral sacral extension would have a falsely negative seated flexion test. However, it has not been specifically associated with childbirth. Answer D


33| A patient presents with right forearm and wrist pain after a fall. There are no neurologic deficits. X-ray of the cervical spine and right arm are normal. Physical exam reveals an increased carrying angle on the right, the hand and wrist are adducted and the olecranon process prefers medial glide. The most likely diagnosis is:

A) Posterior radial head dysfunction
B) Adduction dysfunction of the ulna
C) Abduction dysfunction of the ulna
D) Anterior radial head dysfunction
E) Medial epicondylitis

Answer

Answer C

This is the presentation of an abduction dysfunction of the ulna.

A posterior radial head dysfunction would cause the forearm to prefer pronation, and the radial head would resist anterior glide. This dysfunction is caused by falling on a pronated forearm. The carrying angle is not affected in radial head dysfunctions. Answer A

An adducted dysfunction of the ulna would have a decreased carrying angle, the hand and wrist would be abducted and the olecranon process would prefer lateral glide. Answer B

An anterior radial head dysfunction would cause the forearm to prefer supination and the radial head would resist posterior glide. Falling backward on a supinated forearm can cause this dysfunction. Answer D

Medial epicondylitis (also known as golfer’s elbow) is a strain of the flexor muscles of the forearm near the medial epicondyle. It is associated with pain at the medial epicondyle; it is not associated with a change in the carrying angle. Answer E


34| A patient complains of right leg pain for two days. The patient noticed the pain while running; the pain is located on the leg just below the knee joint on the lateral side. The right fibular head resists anterior springing; the left foot dorsiflexes more than the right. The most likely diagnosis is:

A) Anterior talar dysfunction on the left
B) Dorsiflexion dysfunction of the talus on the right
C) Anterior fibular head dysfunction on the left
D) Posterior fibular head dysfunction on the right
E) Posterior fibular head dysfunction on the left

Answer

Answer D

In a posterior fibular head dysfunction the foot will prefer supination and the proximal fibular head will resist springing anteriorly. Since there is pain in the right leg, pain can be used as the indicator of laterality or sidedness of the dysfunction in the extremities.

An anterior talar dysfunction is associated with resistance to posterior glide, a foot that prefers plantar flexion and has restriction to dorsiflexion. Answer A

An posterior talar dysfunction is associated with resistance to anterior glide, a foot that prefers dorsiflexion and has restriction to plantar flexion. Answer B

In an anterior fibular head dysfunction the foot prefers pronation. The proximal fibular head will resist springing posteriorly. In addition, the patient’s history does not indicate a problem with the left foot. Answer C

In a posterior fibular head dysfunction the foot will prefer supination. The proximal fibular head will resist springing anteriorly. The patient had pain in the right leg, indicating it was a right-sided dysfunction, not left. Answer E


35| A patient with dysmenorrhea would like osteopathic treatment for relief of her pain. Which of the following statements concerning the autonomic nervous system effects on the uterus are true?

A. Parasympathetic stimulation results in constriction of the uterine fundus.
B. Parasympathetic stimulation results in relaxation of the cervix.
C. Parasympathetic stimulation results in relaxation of the uterine fundus.
D. Sympathetic stimulation results in relaxation of the uterine fundus.
E. Sympathetic stimulation results in constriction of the cervix.

Answer

Answer C

Parasympathetic stimulation of the uterus causes relaxation of the fundus and constriction of the cervix. Answers A and B

Sympathetic stimulation of the uterus causes constriction of the fundus and relaxation of the cervix. Answers D, and E


36| A male complains of right foot pain for several weeks. He remembers having the pain before. The pain is located between the third and fourth metatarsal heads. The pain is described as 8/10 in intensity, and burning in nature. What is the most likely diagnosis?

A) Hammer toes
B) Claw toes
C) Morton’s neuroma
D) Bunion
E) Corns

Answer

Answer C

Morton’s neuroma is a fibroneuromatous reaction between the heads of the third and fourth metatarsals. Pain is present at the forefoot at the site of the neuroma it is often accompanied with dysesthesia or burning plantar pain.

A hammer toe is a condition in which the proximal interphalangeal (PIP) joint is hyperflexed. Typically there is an obvious deformity and the patient may have pain at the PIP when wearing shoes. Answer A

Claw toe is a fixed flexion deformity of the PIP joints associated with hyperextension of the metatarsophalangeal articulations. All toes, especially the lesser toes, tend to be effected. Answer B

A bunion, also called hallux valgus, is a lateral deviation of the proximal phalanx of the first toe associated with soft tissue changes, pain, swelling, and inflammation at the aspect of the head of the first metatarsal, which is angled medially. Answer D

Soft corns are hyperkeratotic lesions found between the toes, usually the fourth and fifth toes; they are extremely painful. Hard corns are associated with hammer or claw toes. Answer E


37| A 30-year-old patient with new onset left-sided low back pain comes into your office. Pain stems from the low back and radiates to the left knee. Structural examination reveals the following:
– A tender point located at the right transverse process of L5
– L5 neutral, rotated right and sidebent left
– Positive seated and standing flexion test on the right
– Deep right sacral sulcus, Left inferior lateral angle is posterior and inferior
– Springing at the lumbosacral junction is present
– PSIS on the right is caudad compared to the left PSIS with the patient in the prone position
– Apparent shorter leg on the right

What is the most likely innominate diagnosis?
A) Right innominate anterior
B) Right innominate posterior
C) Right superior innominate subluxation
D) Right inferior innominate subluxation
E) There is not enough information in the question to differentiate from the above

Answer

Answer B

In a right posterior innominate the standing flexion test is positive on the right, the right ASIS is cephalad, the right PSIS is caudad and there is a relative short leg on the right.

In an anterior rotated innominate the PSIS on the right would be cephalad compared to the left PSIS. Answer A

In a right superior innominate subluxation (a.k.a. innominate upslip, superior innominate shear) the right leg would be shorter and the right PSIS would be cephalad. Answer C

In a right inferior innominate subluxation (a.k.a. innominate downslip, inferior innominate shear) the right PSIS would be caudad and the right leg would appear longer. Answer D

There is enough information provided to choose one of the above diagnoses. Answer E


38| What is the accepted muscle energy treatment patient position for the above sacral diagnosis?

A) Right lateral Sims position
B) Right lateral recumbent with the torso turned left
C) Left lateral Sims position
D) Left lateral recumbent with the torso turned right
E) Patient prone with right lower extremity slightly abducted

Answer

Answer C

The patient in the above question has a left on left sacral torsion. The accepted muscle energy treatment position of the patient is the left lateral Sims position.

In this position the patient is lying in the left lateral recumbent position and the torso is turned to the left so the patient is lying face down.

The right lateral Sims position would be used to treat a right on right sacral torsion. Answer A

The right lateral recumbent position with the torso turned left would be used to treat a left on right sacral torsion. Answer B

The left lateral recumbent position with the torso turned right would be used to treat a right on left sacral torsion. Answer D

This position could be used to treat a sacral shear. Answer E


39| The correct typical counterstrain position to treat the above tenderpoint is:


A) Lumbar flexion
B) Lumbar extension
C) Left leg adduction
D) Left leg abduction
E) Left lumbar sidebending

Answer

Answer B

According to Yates the typical treatment position for a tender point at the transverse process of L5 is lumbar extension, sidebending away with rotation either toward or away. Jones just specifies lumbar extension


40| A 65-year-old obese diabetic male comes to the emergency room with severe shortness of breath. He appears slightly anxious. He denies any chest pain. Dyspnea worsens when he lies flat on his back. Dyspnea improves when he is placed in a semi-Fowlers position. He admits to a recent 20lbs weight gain, he has developed swelling of the legs, and has nocturia. He has a history of two myocardial infarctions. On examination, he is 5 foot 8 inches tall and weighs 290
pounds. BP 150/80, HR 105, RR 32 and shallow, Temp. 98.60. On auscultation, there is an 83 gallop rhythm, rales are present in the lower lung fields, abdominal examination reveals a soft and enlarged liver and grade 4 pretibial edema. EKG demonstrates normal sinus rhythm with no changes when compared to his previous hospital admission’s EKG 2 months ago. Which of the following is the most appropriate for initial management of this patient’s symptoms in the emergency room?

A) Intravenous furosemide
B) Oxygen, intravenous fluids and cardiac monitoring
C) Oxygen and digitalis
D) Intravenous tissue plasminogen activator
E) Placement of a 8wan-Ganz Catheter

Answer

Answer A

Treatment of acute congestive heart failure includes oxygen and IV furosemide. Nitroglycerine, which also can be used, is a venodilator that can potentiate the effect of furosemide. Other agents include morphine (reduces anxiety and dilates pulmonary and systemic veins) and nitroprusside (a useful adjunct in the treatment of CHF due to acute valvular regurgitation or hypertension).

Although oxygen and cardiac monitoring would be wise choices, additional intravenous fluids will worsen his congestive heart failure, the goal of initial treatment would be to remove excess fluid. Answer B

Digitalis is useful in the long-term management of heart failure. It is not typically used in the treatment of acute congestive heart failure. Answer C

Thrombolytic therapy (tissue plasminogen activator) is not used in congestive heart failure. Answer D

Placing a Swan-Ganz catheter may be helpful in cases in which a prompt response to therapy does not occur. Answer E


41| Treatment of which area would most likely decrease hypersympathetic tone to this patient’s heart?


A) Occipito-atlantal joint
B) C4
C) T2
D) T6
E) T7

Answer

Answer C

The sympathetic innervation to the heart generally stems from T1 – T5. Treatment to this area may decrease sympathetic tone in this patient.

The OA (atlanto-occipital) joint has a direct influence on the function of the vagus nerve (parasympathetic innervation to the heart). Therefore treatment of the OA (atlanto-occipital) joint may alter parasympathetic tone to the heart. Answer A

Manipulation of the cervical spinal segments near the cervical chain ganglia may influence sympathetic tone to head and neck structures, not the heart. Answer B

Although some texts list T6 as a level for sympathetic influence to the heart, not all texts agree. However, they all do agree on T2, therefore T6 is not the best answer. Answer D

T7 is not considered to have sympathetic influence to the heart. Answer E


42| In this patient. treatment of which area would have the greatest effect on renal function?


A) Occipito-atlantal
B) T5
C) T8
D) T11
E) The sacrum

Answer

Answer D

The kidneys receive sympathetic innervation from segments T10 – T11.
Therefore, treatment of these segments would be most likely to decrease excessive sympathetic tone to the kidneys. This will improve the glomerular filtration rate and increase urinary output.

Although the occipito-atlantal joint will influence the vagus nerve, it is not known how the parasympathetic nervous system affects the kidney. Answer A

The kidney does not receive sympathetic innervation from T5 or T8. Answers B and C

Although treatment of the sacrum will influence the pelvic splanchnic nerve, the kidney does not receive innervation from the pelvic splanchnic nerve. Answer E


43| Which of the following lymphatic techniques listed would be the most appropriate initial treatment in this patient?


A) Liver pump
B) Abdominal pump
C) Classic thoracic pump
D) Thoracic inlet release
E) Pedal pump

Answer

Answer D

As a general rule, it is best to release any diaphragms, especially the thoracic inlet, prior to starting any pumping techniques. Once the diaphragms are free of restriction and sympathetic tone is normalized, lymphatic pumps will help return lymph through open channels.


44| A 42-year-old female with severe rheumatoid arthritis comes to your office with neck pain. She has a history of chronic neck pain, however recently her pain has increased. She denies any trauma and does not recall any specific event that would cause her neck pain to increase. On examination, she has severely diminished range of motion. Palpation of her cervical paraspinals reveals areas of fibrotic tissue with some tenderness noted in the sub-occipital region. The third cervical segment is sidebent left. Neurologic examination is normal. Which of the following describes an appropriate osteopathic technique for C3?

A) C3 is sidebent to the left and rotated to the right against the restrictive barrier, the patient then rotates to the left against the physician’s counter force for 3-5 seconds, then the physician passively rotates the cervical spine farther to the left.
B) C3 is sidebent and rotated to the right against the anatomic barrier, the patient then rotates to the left against the physician’s counter force for 3-5 seconds, then the physician passively rotates the cervical spine farther to the right.
C) C3 is sidebent and rotated to the right against the restrictive barrier, the patient then rotates to the left against the physician’s counter force for 3-5 seconds, then the physician passively rotates the cervical spine farther to the right.
D) The patient’s neck is sidebent to the left and rotated to the left, a high velocity, low amplitude right rotational thrust is applied to the third cervical segment.
E) The patient’s neck is sidebent and rotated to the right, a high velocity, low amplitude right sidebending thrust is applied to the third cervical segment.

Answer

Answer C

This is an example of direct muscle energy using post-isometric relaxation technique.

Sidebending and rotation in the third cervical segment are always toward the same side (type II mechanics). When positioning a patient for a muscle energy technique, C3 should be sidebent and rotated toward the same side. Answer A

C3 is side bent and rotated to the right against the restrictive (not anatomic) barrier, the patient then rotates to the left against the physician’s counter force for 3-5 seconds, then the physician passively rotates the cervical spine farther to the right. Answer B

Since the patient has severe rheumatoid arthritis HVLA is relatively contraindicated and therefore it is not the most appropriate technique. In addition, the patient is not correctly positioned for an HVLA technique in answers D and E.


45| What is the theorized neurophysiologic mechanism by which the correct answer in the above question achieves its effect?

A) The Golgi tendon organs senses a change in tension from the isometric muscle contraction, this causes a reflex relaxation of the agonist muscle fibers. Therefore, by reflex relaxation of the agonist muscle, the physician is then able to passively stretch the patient to the new restrictive barrier.
B) Contracting the agonist muscle causes local prostaglandin release; this produces a localized muscle relaxation and analgesia.
C) Contracting the antagonistic muscle, signals are transmitted to the spinal cord and through the reciprocal inhibition reflex arc; the agonist muscle is then forced to relax.
D) An HVLA thrust is thought to forcefully stretch a contracted muscle producing a barrage of afferent impulses from the muscle spindles to the central nervous system. The central nervous system reflexively sends inhibitory impulses to the muscle spindle to relax the muscle.
E) An HVLA thrust is thought to forcefully stretch the contracted muscle pulling on its tendon, activating the Golgi tendon receptors and reflexively relaxing the muscle.

Answer

Answer A

This correctly describes the neurophysiologic mechanism of the post-isometric relaxation type of muscle energy.

Prostaglandins are not released with active muscle contraction and do not result in local muscle relaxation. Prostaglandins typically cause pain and inflammation; some anti-inflammatory medications block the release of prostaglandins. Answer B

This correctly describes the neurophysiologic mechanism of the reciprocal inhibition type of muscle energy, not the post-isometric relaxation type of muscle energy. Answer C

There are two theories by which HVLA is thought to achieve its effects. They are described by choices D and E. Answers D and E


46| In addition to osteopathic treatment, patient education, and a short course of physical therapy, you decide to prescribe a non-steroidal anti-inflammatory medication. A significant concern regarding long-term use of this medication is:

A) Osteoporosis
B) Upper gastrointestinal bleed
C) Diverticulitis
D) Irreversible hepatocellular toxicity
E) Renal failure

Answer

Answer B

NSAID-associated gastropathy is one of the most prevalent serious drug toxicities in the United States. Approximately 1 – 2% of patients using NSAIDS for three months and approximately 2 – 5% of patients using NSAIDS for one year have gastrointestinal ulcers, bleeding or perforation.

NSAID use has not been linked to osteoporosis or diverticulitis. Although long term use of corticosteroids will lead to osteoporosis. Answers A and C

NSAIDS have been linked with reversible hepatocellular toxicity. Liver enzymes usually revert to normal after the drug is discontinued. As a result, it is recommended that liver enzymes are checked periodically in patients on long term NSAIDS. Answer D

NSAIDS may increase serum creatinine concentrations in some patients. However, renal failure is uncommon in patients without underlying renal impairment. Answer E


47| Complications from the adverse effect in the previous question can be avoided by?


A) Yearly bone densiometry
B) Concomitant use of misoprostol
C) Yearly colonscopy
D) Frequent monitoring of liver enzymes
E) Frequent monitoring of blood urea nitrogen and serum creatinine

Answer

Answer B

In a double blind, randomized, placebo-controlled study 51 misoprostol decreased serious gastrointestinal complications by 40%.

Since NSAID use has not been linked to osteoporosis or diverticulitis, yearly bone densitometry or yearly colonoscopy will not prevent NSAID induced complications. Answers A and C

NSAIDS have been associated with reversible (not irreversible) hepatocellular toxicity and renal failure is uncommon; therefore these are not the best answers.

However, it is recommended that a complete blood count, liver function tests, renal function as well as stool guiac testing be done every 4 months in patients on long term Answers D and E


48| A 51-year-old nurse complains of paresthesias in her left hand. She describes intermittent numbness that worsens while sleeping. Ibuprofen improves her symptoms. She has difficulty handling small objects and buttoning shirts. On examination, she has decreased sensation on the palmar aspect of her second and third digit when compared to the fifth digit. There is mild atrophy of the left thenar eminence. Tender Points are present in the left anterior scalene. The first rib on the left has limited exhalation movement. The lower cervical segments are restricted in left sidebending. The primary reason for treating the cervical somatic dysfunction in the above patient is?

A) To remove possible sites of additional neurologic compression and improve brachial plexus function
B) To improve lymphatic drainage from the upper extremity thus decreasing distal nerve compression
C) To improve vascular drainage from the upper extremity thus decreasing distal nerve compression
D) To decrease sympathetic tone in the upper extremity
E) Due to the distal site of injury in this patient, treatment of the axial spine will not further decrease this patient’s symptoms.

Answer

Answer A

The above patient has carpal tunnel syndrome. Osteopathic treatment for carpal tunnel syndrome includes:

1. Treating rib and upper thoracic somatic dysfunctions to decrease sympathetic tone in the upper extremity.

2. Treating cervical somatic dysfunctions and myofascial restrictions to enhance brachial plexus function and remove potential sites of additional compression.

3. Treating the carpal tunnel with direct release techniques to increase the space in the carpal tunnel.


49| Treatment to which structure will result in the greatest reduction of excessive autonomic tone in the upper extremity


A) The occiput
B) C3
C) C7
D) T3
E) T9

Answer

Answer D

The above patient has carpal tunnel syndrome (CTS). This condition is due to a compression of the median nerve, usually at the transverse carpal ligament. However, cervical or rib dysfunction as well as anterior scalene hypertonicity may further complicate CTS. Ipsilateral upper thoracic and rib dysfunction will result in increased upper extremity sympathetic tone producing arm and hand symptoms. Decreasing sympathetic tone is an integral part of OMT treatment for CTS.

The upper thoracic segments (specifically T2 – T8) supply sympathetics to the upper extremity.

Sympathetic innervation arises from the thoracic and lumbar regions.

Treatment of the occiput, C3, and C7 will not alter the sympathetic tone in the upper extremity. Answers A, B and C

T9 is a segment that generally receives sympathetic fibers from structures below the diaphragm. Answer E


50| Which of the following studies is considered the gold standard for diagnosis of the above condition?


A) Magnetic resonance imaging
B) Arteriography
C) Plain x-ray
D) Nerve conduction studies/Electromyography
E) Bone scan

Answer

Answer D

Nerve conduction studies/electromyography is considered to be the gold standard for the diagnosis of carpal tunnel syndrome.

Nerve conduction studies will identify if there has been any damage to the myelin or axon of the median nerve.

Electromyography will identify if this damage is severe enough to cause denervation of the distal muscles innervated by the median nerve.

Other studies listed have essentially no role in the diagnosis of carpal tunnel syndrome. Answers A, B, C and E



51| A 34-year-old athletic male presents to your office complaining of low back pain. He states that the pain started soon after playing baseball. The pain is localized to his low back. He denies any radiation of pain, paresthesias or weakness in the lower extremities. There is no previous history of low back pain. Physical exam reveals no focal neurologic deficits. X-rays of the lumbar spine are normal. There is marked spasm of the paraspinal musculature on the right. There appears to be a group dysfunction from L1-L5 that is markedly rotated right and sidebent left. There is a positive seated flexion test on the right, and the right sulcus is deeper. Which of the following is the most likely diagnosis?

A) Lumbar spinal stenosis
B) Cauda equina syndrome
C) Acute lumbar strain
D) Herniated nucleus pulposus
E) Spondylolisthesis

Answer

Answer C

An acute lumbar strain is low back pain without focal neurological deficits. The pain may be sharp or dull, and usually localized to the low bacK, although it may radiate in a non-dermatomal type of fashion.

Spinal stenosis is a chronic joint disease characterized by slowly developing joint pain, usually resulting from intervertebral narrowing, foraminal encroachment and/or spinal canal compression. Often this occurs in elderly patients and the neurologic examination will reveal some deficits. Answer A

Cauda equina syndrome is compression of the distal spinal cord. This will result in paresthesias and weakness as well as incontinence. Answer B

A herniated nucleus pulposus (herniated disc) will often result in neurologic deficits therefore making this a less likely diagnosis. Answer D

Spondylolisthesis is a forward slippage of the lumbar vertebrae. Radiographic findings in this patient are normal making this an unlikely diagnosis. Answer E


52| If the patient had reported decreased sensation on the medial aspect of the ankle, absent patellar reflex and marked weakness of ankle dorsiflexion and knee extension, what would the most likely diagnosis be?

A) Spondylolisthesis
B) Lumbar spinal stenosis
C) Herniated nucleus pulposus
D) Acute lumbar strain
E) None of the above are likely diagnoses

Answer

Answer C

Herniated nucleus pulposus. A herniated disc at L4 will result in the decreased sensation, the decreased deep tendon reflexes and lower extremity weakness described.

Radiographic findings on the patient were normal making spondylolisthesis unlikely. Answer A

Spinal stenosis is usually due to intervertebral narrowing and degenerative joint disease. Although neurological deficits are common, spinal stenosis is usually seen in the elderly population. Answer B

Neurologic exam is negative in acute lumbar strains. Answer D


53| If the above patient had a sacral torsion present, based on the above findings what is the most likely diagnosis?

A) Left torsion on Right oblique axis
B) Right torsion on Left oblique axis
C) Left torsion on Left oblique axis
D) Right torsion on Right oblique axis
E) Cannot determine based on the limited information

Answer

Answer C

In sacral torsions, specific L5 findings are present.

L5 will influence the sacrum in the following ways: if L5 is rotated, the sacrum will rotate in the opposite way on an oblique axis.

If L5 is sidebent, the sacral oblique axis is engaged on the same side.

Since the patient’s L5 was NSLRR, the sacrum is rotated left on a left oblique axis.


54| The patient is found to have a tenderpoint 1 cm lateral to the pubic symphysis on the right superior pubic ramus. This tenderpoint correlates to somatic dysfunction of which vertebra?


A) L1
B) L2
C) L3
D) L4
E) L5

Answer

Answer E

Anterior lumbar tender points are located around the pelvis. LS is located approximately 1 cm lateral to the pubic symphysis on the superior rami.

L2, L3 and L4 tender points are located near the AlIS. Answers B, C and 0

L1 tender point is located medial to the ASIS. Answer A


55| Which of the following is the correct treatment position for the above tenderpoint?

A) Patient prone, right knee and hip extended, rotated to contralateral side
B) Patient prone, knees and extended, rotated to ipsilateral side
C) Patient left lateral recumbent position, right knee flexed, hips extended
D) Patient supine, left knee and hip flexed, sidebent and rotated to ipsilateral side
E) Patient supine, both knees flexed and hips flexed to about 90 degrees

Answer

Answer E

An anterior L5 somatic dysfunction is treated with the patient in the supine position.

The knees and hips flexed. Jones reports flexion of both hips to about 135 degrees.

Yates reports flexion of both thighs to 60 – 90 degrees with sidebending and rotation away.



OMM Questions 56-75


56| A 36-year-old female enters your office complaining of hip pain following a fall off of a stepladder 4 days earlier. X-rays taken in the emergency department revealed no fracture, and the patient has been very uncomfortable since the fall. The pain is localized on the left, and does not radiate down the thigh. The left ASIS is cephalad, the left PSIS is caudad and the pubes are level. While performing a standing flexion test, you note that the left PSIS demonstrated greater excursion than the right. Which additional palpatory finding is most likely to be present in this patient?

A) Resistance to posterior compression of the right ASIS
B) Backward sacral torsion on a left axis
C) Longer leg on the right
D) L5 neutral sidebent left, rotated right.
E) Exquisite pain upon palpation of the pubic symphysis

Answer

Answer C

The patient is has a left innominate posterior. A shorter leg on the ipsilateral side (in this case, the left leg) will be present in this dysfunction. Consequently, the contralateral leg (in this case, the right leg) will appear longer.

Resistance to posterior compression is describing the ASIS compression test. This test will confirm the findings of the seated or standing flexion test. Since there is a somatic dysfunction on the left side, posterior compression on the right is likely to demonstrate adequate resiliency (i.e. it will have normal springing motion). Answer A

A backward torsion on a left axis (right on left) is a specific sacral somatic dysfunction and not necessarily associated with an innominate dysfunction. Answer B

L5 neutral, sidebent left, rotated right is also not necessarily associated with a left innominate posterior. Answer D

Exquisite pain upon palpation of the pubic symphysis could be present with a pubic rami fracture or pubic symphysis somatic dysfunction. It is not necessarily present in a left innominate posterior. Answer E


57| What is the innominate somatic dysfunction of the above patient?


A) Left innominate anterior
B) Left innominate posterior
C) Right innominate anterior
D) Right innominate posterior
E) Not enough information to determine

Answer

Answer B

A positive standing flexion test on the left indicates a left innominate problem. The left ASIS is cephalad and the left PSIS is caudad, these findings are indicative of a posterior innominate on the left.

In a left innominate anterior, the left ASIS would be caudad and the left PSIS cephalad. Answer A

In a right innominate anterior or a right innominate posterior, the standing flexion test will be positive on the right. Answers C and D


58| Which of the following soft tissue dysfunctions is the most likely cause of this innominate dysfunction of the above patient?


A) Tight quadriceps
B) Tight hip adductors
C) Tight hip abductors
D) Tight hamstrings
E) Tight piriformis

Answer

Answer D

The most common cause of a left innominate posterior is tight hamstrings. The hamstrings attach to the ischial tuberosity and contraction can cause the innominate to rotate posteriorly.

The most common cause of a left innominate anterior is tight quadriceps. The quadriceps have attachments to the AIlS and contraction can cause the innominate to rotate anteriorly. Answer A

Hip adductors are attached to the pubic bones and femur. Tight hip adductors can cause an inferior pubic shear. Answer B

Tight hip abductors and piriformis tightness has not been specifically linked with innominate dysfunction. Answers C and E


59| Which of the following is the best statement regarding typical muscle energy treatment for this innominate dysfunction?

A) Hip flexors are isometrically contracted to provide the necessary force to correct this dysfunction.
B) The patient is best treated in a seated position.
C) The patient is treated with her hip flexed and slightly abducted
D) Hamstrings are activated by the patient to correct the dysfunction
E) Muscle energy treatment is relatively contraindicated in the above patient

Answer

Answer A

Hip flexors are used to correct the dysfunction. The treatment position to correct a left innominate posterior is as follows: With the patient supine, drop the left leg off the table to engage the restrictive barrier. Instruct the patient to flex their hip against your counterforce for 3-5 seconds.

In the typical muscle energy treatment, for this dysfunction the patient should be supine, not seated. Answer B

Hamstrings are hip extensors and would be used to correct an anterior innominate dysfunction. Answer D

Since the patient is not critically ill, muscle energy is not contraindicated in this patient. Answer E


60| If a right-sided psoas syndrome is suspected in this patient, which lumbar dysfunction is likely to be present?

A) L1 Neutral sidebent left and rotated right
B) L2 Extended, sidebent left and rotated left
C) L1 Flexed, sidebent right and rotated right
D) L2 Flexed, sidebent left and rotated left
E) L4 Neutral sidebent right rotated left.

Answer

Answer C

In psoas syndrome a high lumbar dysfunction is likely to be present. Specifically the lumbar dysfunction will be flexed, sidebent and rotated to the side of the tight psoas. In this case, L1 would be flexed, rotated right and sidebent right.


61| You are examining a 32-year-old male presenting to the emergency room complaining of right-sided flank pain. The pain is intermittent, excruciating and radiates to the right testicle.

  • Physical exam reveals:
    • Vitals: Temp: 98.6F, Heart Rate: 105 Respiratory Rate: 20 Blood pressure 140/90 Pain: 10/10
    • Abdomen: There is no rebound tenderness or signs of peritoneal irritation. There is costovertebral angle tenderness on the right
  • Laboratory data:
    • Urinalysis: RBC’s: too numerous to count, small leukocytes, nitrite negative. The CBC is normal.
  • Radiological data:
    • KUB demonstrates a 6 mm well circumscribed radio-opaque mass approximately 10 cm superior to the middle of the right pubic ramus. IVP confirms the presence and location of the stone with severe hydronephrosis

Which of the following is true regarding osteopathic treatment for the above patient?
A) All osteopathic treatment is absolutely contraindicated in the above patient, until the patient’s pain is controlled.
B) Due to the patient’s excruciating pain, osteopathic treatment should be limited to gentle techniques such as rib raising and paraspinal inhibition only.
C) High velocity thrust to the thoracoLumbar junction will decrease ureterospasm via a CNS inhibitory reflex.
D) Pain would decrease with the treatment of a posterior tender point.
E) Post-isometric relaxation muscle energy is indicated

Answer

Answer B

Initial management of ureterolithiasis includes rib raising and paraspinal inhibition to decrease hypersympathetic tone. Other OMT should be delayed until at least partial pain management has been achieved.

Not all osteopathic treatment is contraindicated in the above patient provided standard medical protocols are followed. Answer A

High velocity thrust and post isometric relaxation muscle energy is contraindicated in the above patient, due to the nature of the illness and level of patient’s pain. Answers C and E

Treatment of a posterior tenderpoint is likely to worsen this patient’s pain, because it typically requires the patient to be in extension. In addition, treating the cause of the pain (ureteral stone) would decrease the patient’s pain, not just counterstrain. Answer D


62| Which of the following soft tissue dysfunctions has been specifically associated with ureterolithiasis?


A) Psoas spasm
B) Abdominal rectus spasm
C) Piriformis tenderpoint
D) Paraspinal spasm
E) Quadratus lumborum spasm

Answer

Answer A

Due to the close proximity of the ureter to the psoas muscle, ureterospasm has been associated with iliopsoas dysfunction and psoas syndrome.

Abdominal rectus spasm and a piriformis tenderpoint are not likely due to the fact that these muscles are not local to the site of irritation. Answers B and D

Paraspinal muscle spasm may be seen in patients with ureteral stones, especially patient’s with costovertebral angle tenderness, however paraspinal muscle spasm can occur in many different dysfunctions and has not been specifically associated with ureterospasm. Answer C

The quadratus lumborum lies posterior to the iliopsoas and would less likely be affected with a ureteral stone. Answer E


63| What is the primary reason for treating the thoracoLumbar junction in the above patient?

A) To decrease this patients pain
B) To increase function of the contralateral kidney
C) To decrease ureterospasm and increase glomerular filtration rate
D) To decrease bladder spasm
E) To decrease this patient’s blood pressure

Answer

Answer C

Ureterolithiasis is associated with ureterospasm. In addition, patients with unilateral renal disease associated with ureteral stones have a decreased glomerular filtration rate. This is probably because hypersympathetic tone to the kidneys will decrease GFR.

Treating the thoracic spine will decrease sympathetic tone to the kidneys (Tl0 – Tll) and treating the lumbar region will decrease sympathetic tone to the ureters associated with ureterospasm.

Although controlling the patient’s pain is important, pain management is usually controlled with medication; osteopathic treatment should be limited to the reduction of sympathetic tone. Paraspinal inhibition and rib raising have been suggested techniques. Answer A

Treatment to the thoraco-Iumbar junction cannot increase function of the normally functioning contralateral kidney. Answer B

Since the obstruction is proximal to the bladder, spasm is not the primary concern in the above patient; therefore it is not the primary reason for treating the thoraco-Iumbar junction. Answer D

Although the patient does have some high blood pressure, it is a result of the patient’s diagnosis. Treating the thoraco-Iumbar junction to decrease the blood pressure is treating a symptom, not the cause. Answer E


64| What is the most likely composition of the mass in the KUB?


A) Struvite
B) Uric acid
C) Calcium oxalate
D) Cystine
E) Bile pigment

Answer

Answer C

Approximately 80% of all renal stones are calcium oxalate and can be visualized on KUB x-ray, 5% are uric acid and 1% is cystine, the remainders are magnesium, ammonium phosphate or calcium phosphate.

Also, uric acid and cystine stones are not easily visualized on a KUB.


65| A 68-year-old-male with COPD is complaining of right shoulder and arm pain. The patient has noticed that the pain gets worse during exacerbation of his COPD, especially, when he has difficulty breathing. Physical examination reveals bilateral rhonchi with decreased breath sounds. Weakness and paresthesias of the right arm and a diminished right radial pulse occur when the right arm is abducted, extended and the patient turns his head to the right side. There is hypertrophy of respiratory accessory muscles and no lymphadenopathy noted. EKG showed a right axis deviation, low voltage complexes, and poor Rwave progression. Chest, cervical spine and right shoulder X-rays are negative.

The most likely diagnosis is:
A) Myocardial infarction
B) Cervical disk herniation
C) Cervical radiculopathy
D) Pancoast’s tumor
E) Thoracic outlet syndrome

Answer

Answer E

The patient has thoracic outlet syndrome. The patient has a classic presentation of this syndrome. This is characterized by weakness and paresthesias with a decreased radial pulse when the arm is abducted, extended when the patient takes a deep breath and turns his head towards the effected shoulder (Adson’s test)

Myocardial infarction is unlikely since acute changes, such as ST elevation or depression are not present in the EKG. The EKG changes in the question are associated with cor pulmonale. COPD is the most common cause of cor pulmonale. Answer A

Cervical disc herniation and radiculopathy would result in weakness and paresthesias in the upper extremity, however it will not result in a decreased peripheral pulse. In addition, C-spine x-rays are negative; therefore radiculopathy from cervical stenosis is unlikely. Answers B and C

Pancoast’s tumor is located at the pulmonary apex. The tumor may cause shoulder and arm pain with paresthesia, however, the chest x-ray would show a density in the right upper lobe. Answer D


66| The most likely etiology of the pain is:

A) Cervical rib
B) Ischemic myocardium
C) Hypertrophied scalene muscles
D) Herniated nucleus pulposus
E) Shoulder dislocation

Answer

Answer C

Based on the history, the most likely etiology is hypertrophied scalene muscles. One of the areas that causes neurovascular compression of the upper extremity is between the anterior and middle scalene. Chronic COPD can result in hypertrophy or increased tension of the scalene muscles.

Although cervical ribs can cause thoracic outlet syndrome, it would have been reported on the cervical spine x-ray. Answer A

Ischemic myocardium is associated with a myocardial infarction, not with thoracic outlet syndrome. Answer B

Herniated nucleus pulposus is not an etiology of thoracic outlet syndrome, but is the cause of cervical radiculopathy. Answer D

Shoulder dislocation is not an etiology of thoracic outlet syndrome and was ruled out by the shoulder xray report. Answer E


67| The most appropriate treatment for the pain would be:

A) Muscle energy treatment of the first rib
B) HVLA treatment of the cervical rib
C) Reduction of the shoulder
D) Counterstrain treatment of the scalene muscles
E) Morphine and nitrates

Answer

Answer D

Counterstrain of the hypertrophied scalene would cause reduced compression on the neurovascular bundle, and is the appropriate treatment.

Although the patient may have a rib 1 dysfunction, the primary etiology of this patient’s symptoms is due to scalene hypertrophy. Therefore treatment of rib 1 is not the best answer. Answer A

Since this patient does not have a cervical rib this would not be the best answer. In addition, a 63 year-old COPO patient may have osteopenia, which would be a contraindication for HVLA. Answer B

Reduction of the shoulder is the treatment for a shoulder dislocation. Answer C

Morphine and nitrates are the treatment for a myocardial infarct. Answer E


68| At what spinal level would you expect to find tissue texture changes related to this patient’s COPD?

A) CO-C2
B) T2-T6
C) T7-T10
D) T11-L2
E) L3-L5

Answer

Answer B

T2-T7 is the spinal level for the sympathetic reflex of the lungs.

Answers A and E have no sympathetic innervation. CO is another term for the occiput.

T7-T10 is the sympathetic innervation for the upper to middle GI tract. Answer C

T11-L2 is the sympathetic innervation for the middle to lower GI tract. Answer D


69| The increased sympathetic tone would cause which of the following in a COPD patient?

A) Thinning of secretions
B) Bronchoconstriction
C) Thickening of secretions
D) Decreased tissue congestion
E) Vasodilatation of the pulmonary vasculature

Answer

Answer C

Increased sympathetic tone in a COPD patient will cause a thickening of pulmonary secretions via increased goblet cell production.

Thinning of secretions is due to an increase in parasympathetic tone or a decrease in sympathetic tone. Answer A

Increased sympathetic tone will also result in bronchodilation, not bronchoconstriction. Answer B

Pulmonary vasoconstriction from increased hypersympathetic tone could result in increased tissue congestion and possibly development of pneumonia. Answer D

Increased sympathetic tone will also result in vasoconstriction, not vasodilation of the pulmonary vasculature. Answer E


70| The mother of a 4-year-old female complains that her child limps when she walks. Originally, she thought that her daughter had foot pain because she had worn new shoes. After a thorough inspection of the feet, the mother found that the pain was located in her left hip. Her daughter has also had a runny nose for seven days without a fever. Physical examination reveals a playful child with a limp that favors the right side. Hips have full range of motion bilaterally but the child guards all motion of the left hip. There are no neurologic deficits noted. Laboratory tests and X- rays of the hip are normal What is the most likely diagnosis?

A) Septic arthritis
B) Legg-Calve Perthes disease
C) Slipped capital femoral epiphysis
D) Developmental dislocation of the hip
E) Transient synovitis of the hip

Answer

Answer E

Transient synovitis of the hip (also known as toxic synovitis) is a nonspecific, common, unilateral (5% bilateral) inflammatory arthritis involving the hip joint, which occurs in children under 10 years of age (typically 3-6 years of age). It is the most common cause of limp with hip pain in children. The male to female ratio is 3-5: 1. There may be a history of a preceding upper respiratory tract infection. Pain may be present in the hip, antero-medial aspect of the thigh and the knee. Occasionally, a low-grade fever of 100-101F may be present. X-ray of the hip is normal.

Septic arthritis is a serious pyogenic infection of the joint space. It occurs most often in children less than 3 years old. The joint is swollen, and effusion, erythema, tenderness, pain and warmth are evident. The child may have a history of a recent bacterial infection. Septic arthritis is usually accompanied by a fever. The knee is the most commonly affected joint followed by the hip, elbow, and
ankle. Answer A

Legg-calve-Perthes disease is a juvenile idiopathic avascular necrosis of the femoral head. The onset is insidious taking weeks to months, which does not fit the case history. Answer B

Slipped capital femoral epiphysis represents a displacement of the femoral head from the femoral neck due to a stress fracture through the femoral capital epiphyseal growth plate. It is classically seen in obese adolescent males. This would have been diagnosed by the X-ray of the hip. Answer C

Developmental dislocation of the hip encompasses the severity spectrum from mild acetabular dysplasia to frank dislocation. Hip X-ray would have demonstrated a shallow acetabulum with a completely or partially subluxed femoral head. Answer D


71| Which treatment will help decrease the symptoms associated with the above condition?


A) Antibiotics
B) Corticosteroids
C) Non-steroidal anti-inflammatory drugs
D) Surgical fixation
E) Hip manipulation

Answer

Answer C

Most children can be treated symptomatically with bed rest at home and non-steroidal anti-inflammatory medication to decrease pain and inflammation.

Antibiotics would be indicated for septic arthritis. Answer A

Corticosteroids are not indicated for any of these conditions. Answer B

Surgical fixation is indicated for slipped capital femoral epiphysis. Answer D

Hip manipulation has not been shown to improve the symptoms or shorten the duration of transient synovitis of the hip. Answer E


72| The most likely etiology for the above condition is:

A) Idiopathic
B) Bacterial
C) Developmental
D) Viral
E) Obesity

Answer

Answer A

The etiology of transient synovitis of the hip is idiopathic, but mild trauma at an age when the socket or acetabulum is not fully developed is a proposed theory.

Bacterial would be the etiology of septic arthritis. Answer B

Transient synovitis of the hip is not a developmental disorder. Answer C

Although one theory proposes a viral etiology for transient synovitis of the hip, Virologic cultures of the effusion have not identified organisms. Therefore viral is not the best answer. Answer D

Slipped capital femoral epiphysis is associated with obese children. Answer E


73| 51-year-old male complains of chronic pyrosis and regurgitation after meals for several years. The patient has been taking antacids and over-the-counter famotidine, but they are no longer relieving his symptoms. The patient admits to drinking five to six beers and four cups of coffee a day, and smoking half a pack of cigarettes a day. These symptoms are typical of:

A) Dyspepsia
B) GERD
C) Peptic ulcer disease
D) Zollinger-Ellison syndrome
E) Mallory Weiss syndrome

Answer

Answer B

This patient has gastroesophageal reflux disease. The typical symptom is heartburn; in this case the disease is refractory to antacids and famotidine. Reclining after eating, chocolate, coffee, caffeine, cigarettes, fatty meals, peppermint, and alcohol worsen the condition.

Dyspepsia is defined as pain or discomfort centered in the upper abdomen. Heartburn (retrosternal burning) should be distinguished from dyspepsia. Patients with dyspepsia often have heartburn as an additional complaint. When heartburn (pyrosis) coupled with regurgitation is the dominant complaint, gastroesophageal reflux disease is nearly always present and should be distinguished from dyspepsia. Answer A

Epigastric pain (dyspepsia) is the hallmark of peptic ulcer disease, which is usually relieved by eating or antacids, with a recurrence of pain two to four hours later. Answer C

Zollinger-Ellison syndrome is caused by gastrin-secreting neuroendocrine tumors (gastrinomas), which result in hypergastrinemia and acid secretion. Over 90% of patients with this syndrome develop peptic ulcers. In most cases, the two conditions are indistinguishable and present as peptic ulcer disease. Answer D

Mallory-Weiss syndrome is characterized by a non-penetrating mucosal tear at the gastroesophageal junction, which is associated with hematemesis. Answer E.


74| Chronic pyrosis and regurgitation would most likely cause paraspinal tissue texture changes from a viscerosomatic reflex at which spinal level?


A) C3-C6
B) T1-T4
C) T3-T8
D) T10-T11
E) L1-L3

Answer

Answer C

Hypersympathetic activity from GERD will result in tissue texture changes at T2-T8 because these segments receive sympathetic innervation from the esophagus, so T3-T8 is the best choice.

C3-C6 have no sympathetic fibers associated with the esophagus. Answer A

Tl-T4 receive sympathetic innervation from the head and neck. Answer B

T10-Tll receive sympathetic innervation from the middle GI tract. Answer D

Ll-L2 have no sympathetic fibers associated with the esophagus. Answer E


75| Treatment of the anterior Chapman reflex point for this patient’s hyperacidity would be directed just lateral to the sternum at the interspace of:

A) Ribs 4 and 5 on the left side
B) Ribs 5 and 6 on the right side
C) Ribs 5 and 6 on the left side
D) Ribs 6 and 7 on the right side
E) Ribs 6 and 7 on the left side

Answer

Answer C

Chapman’s reflex point for hyperacidity is located just lateral of the sternum at the interspace of ribs 5 and 6 on the left side. Overall, this is a very difficult question.

Just lateral of the sternum at the interspace of ribs 4 and 5 on the left side is the Chapman’s reflex point for the left lower lung. Answer A

Just lateral to the sternum at the interspace of ribs 5 and 6 on the right side is the Chapman’s reflex point for the liver. Answer B

Just lateral to the sternum at the interspace of ribs 6 and 7 on the right side is the Chapman’s reflex point for the liver and gall bladder. Answer D

Just lateral to the sternum at the interspace of ribs 6 and 7 on the left side is the Chapman’s reflex point for peristalsis of the stomach. Answer E



OMM Questions 76-100


76| 29-year-old female presents for a routine physical examination. While assessing her posture from the side you observe that an imaginary plum line dropped from the ceiling to the floor would fall posterior to the apex of the coronal suture, through the external meatus, through most of the bodies of the cervical vertebrae, through the shoulder joint, through the bodies of the lumbar vertebrae, just posterior to the axis of the hip, slightly anterior to the axis of the knee joint and slightly anterior to the lateral malleolus. Which of the following best describes this patient’s posture?

A) Military posture
B) Swayback posture
C) Ideal posture
D) Flat back posture
E) Anterior postural deviation

Answer

Answer C

This is a description of ideal posture when viewing a patient from the side.

Military posture, as viewed from the side (in relation to a plumb line) is described as: head tilted slightly posteriorly, cervical curve and thoracic curve normal, anterior cervical and posterior thoracic deviation from plumb line, anterior pelvic tilt, knees extended, ankles plantar flexed. Answer A.

Swayback posture as viewed from the side (in relation to a plumb line) is described as: head forward, cervical spine lordotic, thoracic spine kyphotic, decreased lordosis of lumbar spine, posterior tilt of pelvis, hip and knee joints hyper-extended. Answer B

Anterior postural deviation as viewed from the side (in relation to a plumb line) is described as: entire body leans forward, deviating anteriorly from plumb line, patient’s weight supported by metatarsals. Answer E

Flat back posture as viewed from the side (in relation to a plumb line) is described as: head forward, cervical spine has slightly increased lordosis, thoracic spine slightly kyphotic in upper portion then flattens in lower segments, lumbar lordosis flattened, and hips and knees extended. Answer D


77| A week later, the patient returns with low back pain. Her head is now located forward of the plum line. She has a slight increase in her cervical lordosis. Her thoracic spine is slightly kyphotic in the upper segments with a flattening of the lumbar spine. Her lumbar vertebrae are straightened and have moved posterior relative to the plum line. Which of the following best describes this patient’s new posture?

A) Military posture
B) Swayback posture
C) Posterior postural deviation
D) Flat back posture
E) Anterior postural deviation

Answer

Answer D

Flat back posture as viewed from the side (in relation to a plumb line) is described as: head forward, cervical spine has slightly increased lordosis, thoracic spine slightly kyphotic in upper portion then flattens in lower segments, lumbar lordosis flattened, and hips and knees extended.

Posterior postural deviation as viewed from the side (in relation to a plumb line) is described as: entire body leans backward, deviating posteriorly, balance maintained by anterior thrust of pelvis and hips, and marked lordosis from mid-thoracic spine down. Answer C


78| A patient complains of neck pain. After a thorough exam you find that there are several muscles in spasm. The most painful muscle originates from the transverse processes of the cervical vertebra and inserts onto rib 1. Which muscle is it?

A) Platysma
B) Longus colli
C) Posterior scalene
D) Sternocleidomastoid
E) Anterior scalene

Answer

Answer E

The anterior scalene originates from the posterior tubercle of the transverse processes of C3-C6 and inserts onto rib 1.

Platysma attaches to the fascia and skin of the pectoralis major and the deltoid inferiorly. It attaches to the mandible superiorly. Answer A

Longus colli is found in three parts of the anterior surface of the vertebral column between the atlas and the body of C3. Answer B

The posterior scalene originates from the posterior tubercles of the transverse processes of C4-C6. Its insertion is the external border of the second rib. Answer C

The sternocleidomastoid muscle originates from the mastoid process and the lateral half of the nuchal line of the occipital bone; it inserts at the manubrium and the medial clavicle. Answer D


79| A tender point is located 2cm lateral to the medial end of the left clavicle at the attachment of the inserting neck muscle. What is the counterstrain treatment position for this anterior cervical tender point?

A) Head and neck rotated left, sidebent right
B) Head and neck rotated right, sidebent right
C) Head and neck rotated left, sidebent left
D) Head and neck rotated right, sidebent left
E) Head and neck rotated right, and flexed

Answer

Answer D

The position describes the left seventh anterior cervical tender point. This corresponds to the left sternocleidomastoid muscle (SCM) and it is considered a maverick point. The appropriate treatment position is to rotate away (right), and side bend toward (left).


80| Spasm of the muscle in question 79 could result in which one of the following conditions?


A) 1st rib dysfunction
B) Left cervical rotation
C) Second rib elevation
D) Thoracic outlet syndrome
E) Torticollis

Answer

Answer E

Torticollis results from a shortening of the sternocleidomastoid (SCM) muscle.

A first rib dysfunction or second rib elevation would not likely occur with spasm of the SCM since it does not attach to these structures. Answers A and C

The SCM rotates the head in the opposite direction, therefore it would cause right cervical rotation. Answer B

Spasm of the SCM is not associated with thoracic outlet syndrome. Answer D


81| Further examination of the patient reveals that ribs 2 – 6 lag behind with inhalation. What muscle would be used to treat this dysfunction if you used a muscle energy treatment?


A) Anterior scalene
B) Middle scalene
C) Posterior scalene
D) Pectoralis minor
E) Serratus anterior

Answer

Answer C

In a group exhalation dysfunction, the key rib is the uppermost rib; treatment is directed at this key rib.

The posterior scalene muscle attaches to the second rib, and is used for a muscle energy treatment of an exhalation dysfunction.

The anterior and middle scalenes are used to treat an exhalation dysfunction of the 1st rib. Answers A and B

Pectoralis minor is used to treat an exhalation dysfunction of ribs 3-5. Answer D

Serratus anterior is used to treat an exhalation dysfunction of ribs 6-9. Answer E


82| Which of the following is associated with a right on left sacral torsion?

A) A non-neutral dysfunction of L5 that is rotated left and sidebent left; and a positive seated flexion test on the right
B) A non-neutral dysfunction of L5 that is rotated right and sidebent right; and a positive seated flexion test on the left
C) A neutral dysfunction of L5 that is rotated right and sidebent left; and a positive seated flexion test on the right
D) A neutral dysfunction of L5 that is rotated left and sidebent right; and a positive seated flexion test on the left
E) A flexed dysfunction of L5 that is rotated right and sidebent right; and a positive seated flexion test on the right

Answer

Answer A

A right on left sacral torsion would have an L5 that is non-neutral (meaning either flexed or extended), rotated left, sidebent left with a seated flexion test positive on the right (opposite the axis of the torsion).

A non-neutral dysfunction of L5 that is rotated right and sidebent right and a positive seated flexion test on the left would be associated with a left on right sacral torsion. Answer B

A neutral dysfunction of L5 that is rotated right and sidebent left and a positive seated flexion test on the right would be associated with a left on left sacral torsion. Answer C

A neutral dysfunction of L5 that is rotated left and sidebent right and a positive seated flexion test on the left would be associated with a right on right sacral torsion. Answer D

A flexed dysfunction of L5 that is rotated right and sidebent right and a positive seated flexion test on the right could not be associated with any sacral torsion. Answer E


83| Which sacral region in the above dysfunction will spring freely?


A) Left superior sulcus
B) Right superior sulcus
C) Left inferior lateral angle
D) Right inferior lateral angle
E) Sacral base

Answer

Answer C

Springing is present over the part of the sacrum that has freedom of motion anteriorly. The left inferior lateral angle moves anterior (as the right sacral base moves posterior) and thus will spring freely in a right on left sacral torsion.

The left superior sulcus and the right inferior lateral angle will not spring freely because they are part of the left oblique axis of the torsion. As a general rule, motion (springing) over the parts of the oblique axis is restricted. Answers A and D

The right sulcus is rotated posteriorly. Springing is restricted over the part of the sacrum that has moved posterior. Answer B

In order to perform the lumbosacral spring test, the physician will spring at the lumbosacral junction (this includes the sacral base). The lumbosacral spring test is positive (meaning it does not spring) in a backward sacral torsion. Answer E


84| Contraindicated in acute hepatitis?

A) Dalrymple Pump
B) Galbreath Technique
C) Thoracic Pump
D) Splenic Pump
E) Liver Pump

Answer

Answer E. LIVER PUMP

The liver pump is a lymphatic technique that involves patient cooperation (exhaling and inhaling) upon command, while the practitioner’s hand leans on the thoracic cage applying a vibratory motion. This results in a pumping action in the liver. This is indicated in several conditions, such as congestive heart failure, congestion of the liver, etc. But it is contraindicated in acute hepatitis


85| Increases fluid flow from middle ear structures via the eustachian tube?

A) Dalrymple Pump
B) Galbreath Technique
C) Thoracic Pump
D) Splenic Pump
E) Liver Pump

Answer

Answer B. GALBREATH TECHNIQUE

The Galbreath Technique is a mandibular drainage technique that relaxes the medial pterygoid muscle, which enables the tensor veli palatini muscle to functionally open the eustachian tube. This technique is useful in acute otitis media.


86| Contraindicated in deep venous thrombosis?

A) Dalrymple Pump
B) Galbreath Technique
C) Thoracic Pump
D) Splenic Pump
E) Liver Pump

Answer

Answer A. DALRYMPLE PUMP

The pedal (a.k.a. Dalrymple) pump is a venous and lymphatic drainage technique applied through the lower extremities.

It is contraindicated in patients with deep venous thrombosis.


87| Indicated for pneumonia?

A) Dalrymple Pump
B) Galbreath Technique
C) Thoracic Pump
D) Splenic Pump
E) Liver Pump

Answer

Answer C. THORACIC PUMP

The thoracic pump is a technique that consists of intermittent compression of the thoracic cage, which is indicated for pneumonia and other pulmonary conditions


88| Contraindicated in infectious mononucleosis?

A) Dalrymple Pump
B) Galbreath Technique
C) Thoracic Pump
D) Splenic Pump
E) Liver Pump

Answer

Answer D. SPLENIC PUMP

The splenic pump improves immune function and is indicated in infection and fever, however, it is contraindicated in splenomegaly.

This is a common finding in infectious mononucleosis.


89| Helps decongest the medial pterygoid plexus?

A) Dalrymple Pump
B) Galbreath Technique
C) Thoracic Pump
D) Splenic Pump
E) Liver Pump

Answer

Answer B. GALBREATH TECHNIQUE.

Relaxing the medial pterygoid muscle helps decongest the rich medial pterygoid plexus.


90| A 45-year-old female presents to your office with low back pain. Upon inspection of the lumbar spine, you observe a left lateral convexity. Palpation reveals fullness in the left paraspinal musculature and L2 – L5 have transverse processes posterior on the left. Given the above information the somatic dysfunction is most consistent with:

A) Type II mechanics with the lumbar spine sidebent left, rotated left
B) Type II mechanics with the lumbar spine sidebent right, rotated right
C) Type I mechanics with the lumbar spine sidebent right, rotated left
D) Type I mechanics with the lumbar spine sidebent left, rotated right
E) There is not enough information given to make a correct diagnosis

Answer

Answer C

The patient has a left lateral convexity. In other words, the lumbar spine is sidebent to the right.

You also note on palpation that there is left paraspinal fullness and L2 – L5 have transverse processes posterior on the left (Le. L2 – L5 is rotated to the left).

When the spine sidebends and rotates in opposite directions, we call this type I mechanics. Type I mechanics occur when the spine is in the neutral position (no flexion or extension) and are typical of group dysfunctions.

Type II mechanics occur when the spine is either flexed or extended, in this case sidebending and rotation would be in the same direction. Type II mechanics are typical of single vertebral dysfunctions.


91| In a 46-year-old male with back pain, you observe that thoracic segments T8 – T12 have a convexity to the right. Which of the following is the best statement regarding this group somatic dysfunction?

A) T9’s right transverse process is more anterior
B) T12 is limited in right rotation compared to L1
C) T10’s left transverse process is more caudad compared to its right
D) Ribs 7 – 12 are limited in exhalation on the left
E) T9 is rotated right, sidebent right

Answer

Answer C

If T8 – T12 have a convexity to the right, there is a group dysfunction that is sidebent left and rotated right. Remember group dysfunctions occur in the neutral plane and in such a case they sidebend and rotate to opposite sides. If T1 0 is sidebent left, then its left transverse process will be more caudad than its right transverse process.

If T9 is rotated to the right, then its right transverse process would be more posterior. Answer A

If T12 is rotated right, it will be limited in left rotation. Answer B

A thoracic dysfunction does not always preclude a rib dysfunction. There is not sufficient information given to determine if a rib dysfunction is present. Answer D

The question is describing a neutral dysfunction that is rotated right, sidebent left. Therefore T9 cannot be rotated right and sidebent right. Answer E

 

92| As a sports medicine physician you are consulted on a 21 year-old female with patello- femoral tracking syndrome. In addition to medication and osteopathic treatment you decide to facilitate the patient’s recovery by recommending exercises to strengthen the vastus medialis. You inform her that while in the supine position with the knee fully extended, she should tighten her quadriceps, hold for 5 seconds then relax. Which of the following statements most accurately describes this type of exercise?

A) Eccentric contraction in which there is no increase in vastus medialis tension.
B) Concentric contraction in which there is an increase in vastus medialis tension.
C) Isometric contraction in which there is an increase in vastus medialis tension.
D) Eccentric contraction in which there is a shortening of the muscle.
E) Isotonic contraction in which there is an increase is vastus medialis tension

Answer

Answer C

The patient in the above question is performing a type of isometric contraction. The definition of an isometric contraction is a muscle contraction that results in an increase in tension without a change in length.

Concentric and eccentric contractions are contractions in which there is an approximation (concentric) or lengthening (eccentric) of muscular origin and insertion. Tension is not related to these type of contractions. Answers A, Band D

An isotonic contraction is one in which there is an approximation of the muscle’s origin and insertion without an increase in muscle tension. The difference between concentric and isotonic contractions is that concentric contractions can have variable tensions. Answer E


93| C3 is extended, sidebent left and rotated left. If you decide to use a direct muscle energy technique to correct this dysfunction how would you position C3?

A) In flexion, sidebent right, rotated right
B) In flexion, sidebent left, rotated right
C) In extension, sidebent left, rotated left
D) In extension, sidebent right, rotated right
E) In extension, sidebent left, rotated right

Answer

Answer A

Direct muscle energy treatment would reverse all three planes of C3 (Le. C3 would be placed in such a way that the segment is against its restrictive barrier in all three planes). In order to correctly perform typical muscle energy the segment must be flexed, sidebent right, and rotated right
.
Flexing, sidebending left and rotating right would not reverses all 3 planes of motion. Answer B

Extending, sidebending left, and rotating left will place C3 away from the restrictive barrier. This position would be used for an indirect treatment. Answer C

Extending would not place the segment against its restrictive barrier. Answers D and E


94| An 81-year-old male with atherosclerotic heart disease was admitted to the hospital with an exacerbation of COPD. He eventually developed pneumonia and went into acute respiratory failure. He was intubated and received assisted mechanical ventilation for 2 weeks. He was extubated and transferred to a general medical floor where he spent two additional weeks. Today, he arrives at a skilled nursing facility where you are the covering house physician. On your admission history and physical exam, the patient complains of low back pain. On examination, you notice that this severely debilitated male has generalized muscle atrophy and requires maximal assistance with bed mobility. He is unable to tolerate sitting or standing. You conclude that his back pain is due to his prolonged bedridden state. Which of the following treatment protocols is best suited for this patient?

A) Low velocity moderate amplitude techniques followed by gentle isometric exercises to strengthen the lumbar spine
B) Myofascial release of the sacrum followed by isotonic exercises with low resistance to strengthen the lumbar spine
C) Rib raising to help with respiration followed by muscle energy to the pelvis and lumbar somatic dysfunctions
D) Indirect treatments to the lumbar fascia followed by direct gentle artiCUlatory techniques
E) Myofascial release to the thoraco-Iumbar junction followed by gentle high velocity low amplitude treatment

Answer

Answer D

The elderly patient in the above question is recovering from severe pulmonary complications from his COPD. He has been bedridden for several weeks and he has coronary disease. As noted in the Foundations of Osteopathic Medicine elderly patients and hospitalized patients typically respond better with indirect techniques or gentle direct techniques, such as rib raising.

This patient could not tolerate isometric exercise due to his low vitality. Answer A

The patient will not be able to tolerate isotonic exercises with low resistance, even though myofascial release of the sacrum may help this patient. Answer B

Although rib raising will help the patient’s respiratory status, this patient will not be able to tolerate muscle energy because of his low Vitality. Answer C

Although this patient may tolerate myofascial release, he has been bedridden for several weeks. Thus, his bone mineral density may be decreased to the level of osteoporosis. High velocity low amplitude techniques are thus contraindicated. Answer E


95| In a patient with a history of tension headaches, structural exam reveals the following: C5 resists lateral translation to the left. There is right-sided fullness at C2 and the right articular pillar of C2 resists anterior glide. Given the above information, what else is most likely true regarding physical examination?

A) C2 is flexed
B) C2 is rotated left
C) The atlanto-axial joint is rotated right
D) C5 is rotated left
E) The atlas is rotated left

Answer

Answer D

If C5 resists lateral translation to the left. This indicates that C5 resists right sidebending. If it resists right sidebending, it is sidebent left. If it is sidebent left, it must be rotated left (C5 follows the rules of type II mechanics).

Since the question did not test C2 in flexion and extension, we do not know whether C2 is flexed, extended or neutral. Answer A

A common way to test for rotation in the cervical spine is to push anterior on the articular pillar of the cervical spine at each segment. If there is resistance on one side compared to the other then the segment Is rotated toward that side. In order to test rotation at C2 the practitioner would push anterior on each articular pillar. The above patient has paraspinal fullness on the right and the right articular pillar resists anterior glide, this indicates that C2 is rotated to the right. Answer B

The above question does not describe the direction of the atlantoaxial joint (C1 on C2). Answers C and E


96| Which of the following is true regarding the cervical spine?

A) Hypertrophic changes at the C4/C5 facet joint and degenerative changes at the corresponding joint of Luschka may cause C4 nerve root entrapment
B) The synovium of the uncovertebral joint is continuous with that of the facet joint.
C) The main motion of the OA joint is rotation
D) The joints of Luschka play an important role in cervical sidebending
E) Restriction within the sternocleidomastoid will decrease neck rotation to the contralateral side

Answer

Answer D

The joints of Luschka (also called the uncovertebral joints) are only located in the cervical spine. Most authors agree that they are not true synovial joints, however they play an important role in cervical motion, especially sidebending. They also serve to protect nerve roots from disc herniation.

Since the C5 nerve root exits between the C4 and C5 vertebrae, degenerative changes of the joint of Luschka and hypertrophy of the facet joint may cause C5 nerve root entrapment, not the C4 nerve root. Answer A

If the uncovertebral joint (a.k.a. the joint of Luschka) was a true synovial joint (again most authors think that it is not) its synovium would not be continuous with that of the facet joint. This is because the facet joints and uncovertebral joints are located on opposite sides of the intervertebral foramen. Answer B

The main motion of the OA joint is flexion and extension. Approximately half of the flexion and extension of the C-spine stems from the OA joint. Answer C

Restriction within the sternocleidomastoid muscle will decrease rotation to the ipsilateral side. Answer E


97| Which of the following biomechanical or structural abnormalities best explain why a patient with COPD has an increased AP diameter of his/her chest?

A) Fibrotic changes within the diaphragm due to chronic hypoxia results in a flattened diaphragm and an increased total lung capacity.
B) Continuous accessory muscle use will pull the ribs superiorly, resulting in an expanded thoracic cage.
C) The destruction of elastic fibers and chronic air trapping results in permanent inhaled rib positions.
D) Chronic deoxygenation of peripheral tissues results in a physiologic expansion of the thoracic cage as a compensatory mechanism.
E) Exaggerated sympathetic input from visceral afferents causes broncho-aveolar expansion and results in an increased AP diameter.

Answer

Answer C

Destruction of elastic fibers results from years of smoking. (an emphysema type of COPD). Irritation of the lung parenchyma may result in an overproduction of mucus causing obstruction and air trapping (chronic bronchitis and asthma type of COPD). These two factors expand the chest cavity and can result In an Increased lung capacity, thus permanently positioning the ribs in inhalation.

Patients with COPD will have impaired oxygenation, however it will not result in a fibrosis and flattening of the dIaphragm. If this were true all skeletal muscles in the body would be fibrosed. Answer A

Most accessory muscles will help elevate the ribs into inhalation, however these muscles are not strong enough to overcome the natural recoil of the lungs. Answer B

Chronic deoxygenation will result in a change in minute ventilation, rather than a permanent expansion of the thoracic cage. Answer D

Increased sympathetic tone from visceral afferents results in bronchodilation and consequently bronchoavelolar expansion. However, this is not the primary reason for an increased AP diameter in a person with COPD. Answer E


98| With deep inspiration, ribs 6-10 increase the chest wall diameter


A) Anteriorly along an oblique plane
B) Anteriorly around a transverse axis
C) Laterally around an anterior-posterior axis
D) Laterally around a transverse axis
E) Laterally around a vertical axis

Answer

Answer C

Ribs 6-10 primarily move in a bucket-handle motion. Movement is around an AP axis, therefore, inhalation will increase the transverse diameter of the chest wall.

Physiologic rib motion does not occur along an oblique plane. Answer A

Pump-handle ribs will increase the chest wall anteriorly around a transverse axis. Answer B

Rib movement around a transverse axis could not expand the ribcage laterally. Answer D

Ribs do not move around a vertical axis. Answer E


99| A 34-year-old male presents to your office with severe low back pain. The pain started yesterday while he was working on his car and is now radiating into his lower extremities. What additional information in this patient’s history would most impact the immediate diagnosis and treatment?

A) Absent unilateral superficial abdominal reflex
B) Presence of incontinence
C) Loss of Achilles reflex
D) Down-going plantar response with Babinski’s testing
E) Unilateral loss of cremasteric reflex

Answer

Answer B

A patient with low back pain radiating into BOTH lower extremities may be of neurogenic or musculoskeletal origin. Of all the neurogenic or musculoskeletal possibilities the most important entity to rule out is Cauda Equina Syndrome (CES). CES is an entrapment of terminal nerve roots of the spinal cord. This can be due to a central disc herniation. If S2-S4 nerve roots are involved incontinence can result. If this occurs, immediate surgical decompression is indicated. If decompression is delayed, this may result in irreversible incontinence.

Unilateral loss of the superficial abdominal reflex indicates a lower motor neuron lesion from T7 – L2, depending on where the absence is noted. If a lower motor neuron lesion was suspected, such as a herniated disc, it may impact his diagnosis, but this treatment is still likely to be conservative management. Answer A

If the patient had an absent Achilles reflex, this would indicate a lower motor neuron lesion at S1 If this were present, the treatment would still likely be conservative. Answer C

A downgoing plantar response when trying to elicit a Babinski reflex is a normal adult response. In other words, there is no Babinski reflex therefore, there is no suspected upper motor neuron lesion. However, the patient may still have a LMN lesion present. Answer D

A unilateral loss of the cremasteric reflex suggests a lower motor neuron lesion between L1 and L2. This finding may impact diagnosis and treatment, however it would not impact the diagnosis and treatment as much as it would if it were cauda equina syndrome. Answer E


100| A posterior fibular head dysfunction can cause direct compression on which of the following peripheral nerves?


A) Common fibular
B) Obturator
C) Sciatic
D) Tibial
E) Sural

Answer

Answer A

The common fibular nerve (a.k.a. common peroneal nerve) courses behind (posterior to) the fibular head. A posterior fibular head dysfunction can exert pressure directly on the common fibular nerve resulting in foot drop.

Fibular head dysfunction will not directly compress the obturator or sciatic nerves. Answers B and C

The tibial nerve courses through the popliteal fossa, it is not affected by a fibular head dysfunction. Answer D

The sural nerve is made up of fibers from the common peroneal and tibial. It would not be compressed in fibular head dysfunction since these branches arise proximal to the fibular head. Answer E


Popular Links


Best Residency Interview Questions